Download as pdf or txt
Download as pdf or txt
You are on page 1of 162

‫"غير مسموح السرقة منه ‪ ،‬وهو خاص بطالب استاذ محمد‬

‫سامي‬
‫الملف األن امانه وفي ذمتك ويمنع إرسال الملف خارج القروب"‬

‫تجميع مارس ‪2019‬‬

‫تجميعات‬
‫‪T. MOHAMMED SAMI‬‬
‫أفنــان العنزي ‪،‬إقبـال ‪،،‬عبد املجيد الشمري‪ ،‬فاطمه‪ ،‬فوز الشهراني ‪ ،‬نوره‬

‫الصفحة ‪1‬‬ ‫استاذ ‪ :‬محمد سامي جوال ‪0569945982 /‬‬


Parametric exam collection 2019

Maternity

1)caring for a 3 day old neonate who is receiving phototherapy to


treat jaundice. The nurse in charge Would expect to do which of
the following?

‫ الممرضة‬.‫ أيام يتلقى عالجًا ضوئيًا لعالج اليرقان‬3 ‫) عند رعاية طفل حديث الوالدة عمره‬1
‫المسؤولة ماذا تتوقع أن تفعل مما يلي؟‬

A- turn the neonate every 6 hours

B- encourages the mother to discontinue the

C- notify the physician if the skin becomes bronze in color

D- check the vital signs every 2 to 4 hours

‫اإلجابة‬D

‫نقوم بمراقبة العالمات الحيوية كل ساعتين او اربع ساعات‬

2 ‫الصفحة‬ 0569945982 / ‫ محمد سامي جوال‬: ‫استاذ‬


2) a prim gravida in active labor is about 9 days post term. The
client diseires a bilateral pudendal block anesthesia before ,after
the nurse explains this type of anesthesia to the client. Which of the
following locations identified by the client as the area of relief
would indicate to the nurse that the teaching Was effective?

‫ بعد انتهاء‬pudendal block anesthsia ‫مريضه مخطط لها تخدير إلزالة األلم اسمه‬
‫ المعلومه الصحيحه من قبل الممرضه‬،‫الممرضه من شرح هذا النوع من التخدير للمريضه‬
.‫بخصوص المنطقه التي من المفترض أن ال تشعر فيها بااللم‬

A-back

B-perineum

C-fundus

D-abdomen

3) The first thing that a nurse must ensure when the baby’s head
comes out is?

‫التصرف األولى المطلوب من الممرضه القابله التي يجب أن تقوم به عندما يبدأ خروج رأس‬
‫الطفل‬

A-The cord is intact

B-No part of the cord is encircling the baby’s neck

C-The cord is still attached to the placenta

D-The cord is still pulsating

‫بغرض التأكد من سالمة الطفل يجب أن اليكون اي جزء من الحبل السري ملتف حول رقبة‬

‫الطفل‬

3 ‫الصفحة‬ 0569945982 / ‫ محمد سامي جوال‬: ‫استاذ‬


4)The nurse is assessing a pregnant client in the second trimester of
pregnancy who was admitted to the maternity unit with a
suspected diagnosis of abruptio placentae. Which assessment
finding should the nurse expect to note if this condition is present?

‫اي من العالمات االتيه تتوقع وجودها الممرضه لمريضه مشخصه بانفصال المشيمة المبكر‬

(abruptio placenta)?

A-Dark vaginal bleeding

B-bright red vaginal bleeding

C-painless

D-Non of the above

5) a nurse in postpartum unit is caring for a client who has just


delivered a newborn infant following a pregnancy with placenta
Previa. The nurse reviews the plan of care and prepares to monitor
the client for which of the following risks associated with
placenta Previa?

‫أي من االتي يعتبر من المخاطر المتوقعه والمترافقه مع مريضه مشخصه بهبوط المشيمه‬

)placenta Previa(

A-disseminated intravascular coagulation

B-chronic hypertension

C-infection

D-hemorrhage

‫النزيف متوقع بعد الوالده‬

4 ‫الصفحة‬ 0569945982 / ‫ محمد سامي جوال‬: ‫استاذ‬


6) which of the following terms applies to the tiny, Blanched,
Slightly raised end arterioles found on the face, neck, arms, and
chest during pregnancy?
ً‫أي من المصطلحات التالية ينطبق على الشرايين الطرفية الصغيرة والمبيضة قليال‬
‫والموجودة على الوجه والعنق والذراعين والصدر أثناء الحمل؟‬

A-epulis

B-Linea nigra

C-striae gravdiarum

D-telangiectasias

7) a nurse is assessing client in the 4th stage of labor and notice that
the fundus is firm but that bleeding is excessive. The initial nursing
action would be which of the following?

‫التدخل التمريضي المبدئي والمهم لمريضه في المرحله الرابعه من الوالده لديها نزيف‬
‫غزير؟‬

A-massage the fundus

B- place the mother in the Trendelenburg position

C-notify the physician

D-record the findings

8)which of the following is the sign’s may indicate Pregnancy?

‫العالمه األكيده التي تدل ع الحمل؟‬

A-Chadwicks Sign’s

B- turners sign’s

C-virchows’s sign’s

D-kock’s sign’s

5 ‫الصفحة‬ 0569945982 / ‫ محمد سامي جوال‬: ‫استاذ‬


9)A pregnant client is making her first Antepartum visit. She has a
two year old son born at 40 weeks. A 5 year old daughter born at
38 weeks. And 7 year old twin daughters born at 35 weeks. She had
a spontaneous abortion 3 years ago at 10 weeks. Using the GTPAL
format. The nurse should identify that the client is?
ً ‫ أسبو‬40 ‫ لديها ابن عمره سنتين ولد في‬.‫مريضه حامل تقوم بأول زيارة لها قبل الوالدة‬
.‫عا‬
‫ سنوات‬7 ‫ والبنات التوأم البالغات من العمر‬.‫ أسبوعا‬38 ‫ سنوات ولدت في‬5 ‫ابنة عمرها‬
‫ باستخدام‬.‫ أسابيع‬10 ‫ سنوات في‬3 ‫ كان لديها اإلجهاض تلقائي قبل‬.‫ أسبوعا‬35 ‫ولدت في‬
‫يجب على الممرضة تحديد أن المريضه ؟‬GTPAL. ‫تنسيق‬

‫ الخامس اين؟‬G

G1 ‫حامل حاليا تعتبر‬

A-G4 T3 P2 A1 L4

B-G5 T2 P2 A1 L4

C-G5 T2 P1 A1 L4

D-G4 T3 P1 A1 L4

6 ‫الصفحة‬ 0569945982 / ‫ محمد سامي جوال‬: ‫استاذ‬


10)lochia normally disappears after how many days postpartum?

‫طبيعيا تختفي في فترة مابعد الوالده بكم يوم؟‬Lochia

A-5days

B-7-10days

C-18-21 days

D-28-30days

11)29year-old married woman in the Gynecology Word is


diagnosed of endometriosis. She has difficulty in urination and
heavy menstrual flows. The patient wants more understanding of
her Diagnosis?

‫مريضه لديها صعوبه في التبول وغزاره أثناء فترة الطمث شُخصت ببطانة الرحم المهاجره‬
‫تريد أن تفهم اكثر حول حالتها؟‬

A- Bi furcation of uterine cavity

B-Thickening of intra-uterine cavity

C-Genetic disorder of uterine Tissue

D-Extra Uterine tendency to implant Ov

7 ‫الصفحة‬ 0569945982 / ‫ محمد سامي جوال‬: ‫استاذ‬


12)A patient had a total abdominal hysterectomy 2days-ago and
has not been out of the bed yet. The patient is complaining left leg
pain and swelling. What should the nurse do First?

‫مالذي يجب ع‬،‫مريضه بعد عمليه استئصال الرحم تشتكي من ألم وانتفاخ بالساق اليسرى‬
ً‫ه القيام به أوال‬/‫الممرض‬

A-Gently massage the patient’s leg

B-Assess the patient’s pain level

C-Assess the patient for Homan’s sign

D-Instruct the patient to reflex the left knee and hip

13) a nurse is performing an assessment of a client who is


scheduled for a cesarean delivery. Which assessment finding
Would indicate a need to contact the physician?

‫ه تقوم بتقييم مريضه مجدول لها عمليه قيصريه اي من العالمات االتيه تشير‬/‫ممرض‬
‫للحاجه الي التواصل مع الطبيب؟‬

A-fetal heart rate of 180 beats per minute

B- white blood cell count of 12.000

C-maternal pulse rate of 85 beats per minute

D-Hemoglobin of 11. 0g/dl

8 ‫الصفحة‬ 0569945982 / ‫ محمد سامي جوال‬: ‫استاذ‬


14) Which of the following is described as premature separation of
a normally implanted placenta during the second half of
pregnancy. Usually with severe hemorrhage?

‫تعريف انفصال المشيمة المبكر‬

A. Placenta previa

B. Ectopic pregnancy

C. Incompetent cervix

D. Abruptio placentae

15) A nurse is caring for a client in labor. The nurse determines


that the client is beginning in the 2nd stage of labor when which of
the following assessments is noted?

‫تبدأ المرحله الثانيه من الوالده عندما تالحظ الممرضه ماذا؟‬

A-The client begins to expel clear vaginal fluid

B-The contractions are regular

C-The membranes have ruptured

D-The cervix is dilated completely

‫تمدد عنق الرحم بشكل كامل‬

9 ‫الصفحة‬ 0569945982 / ‫ محمد سامي جوال‬: ‫استاذ‬


16)A laboring client is in the first stage of labor and has progressed
6 Cm in cervical dilation. In which of the following phases of the
first stage does cervical dilation occur most rapidly?

‫ماالمرحلة التي يحدث عندها تمدد عنق الرحم بسرعة أكبر؟‬

A-Preparatory phase

B-Latent phase

C-Active phase

D-Transition phase

17)A mother has delivered in 38 weeks and the baby weight was
1800 g What do you think about the weight pattern?

very low weight birth ‫يكون‬1500g ‫اقل من‬

A- Low weight birth

B- Very low weight birth

C-Appropriate to gastional age

D- Small to gastional age

18) A client in labor is transported to the delivery room and is


prepared for a cesarean delivery. The client is transferred to the
delivery room table, and the nurse places the client in the?

‫الممرضه تضع مريضه مجدول لها عملية قيصريه في اي وضعيه؟‬

A-Trendelenburg’s position with the legs in stirrups

B-semi-fowler position with a pillow under the knees

C-prone position with the legs separated and elevated

D-supine position with a wedge under the right hip

‫وضع األستلقاء على الظهر مع اسفنج تحت الفخذ األيمن‬

10 ‫الصفحة‬ 0569945982 / ‫ محمد سامي جوال‬: ‫استاذ‬


19)What type of milk is present in the breasts 7 to 10 daysPP?

‫ ايام‬١٠ ‫الي‬٧ ‫أي نوع من الحليب األتيه تبقى بعد الوالده من‬

A-colostrum

B-Hind milk

C-mature milk

D-transitional milk

20)late deceleration patterns are noted when assessing the monitor


tracing of a women whose labor is being induced with an infusion
of Pitocin. The women is in a side-lying Position and her vital signs
are stable and fall within a normal range, contractions are intense
last 90 seconds and occurs every 1 1/2to 2 minutes . The nurse
immediate action would be?

‫مريضه تتناول محفزات وألن االنقباضات شديده التصرف الضروري والالزم من قبل‬
‫الممرضه يكون؟‬

A-changes the women’s position

B-stop the Pitocin

C-elevate the women’s leg

D-administer oxygen via a tight mask at 8 to 10 liters /minutes

‫يجب أن اوقف المحفز‬

11 ‫الصفحة‬ 0569945982 / ‫ محمد سامي جوال‬: ‫استاذ‬


21)A woman has Polycystic ovary syndrome which is 3 cm in size,
What should the nurse do?

‫ ماذا يجب أن تفعل الممرضة؟‬، ‫ سم‬3 ‫متالزمة المبيض المتعدد الكيسات التي يبلغ حجمها‬

A- repeat the ultrasound after menstruation

B- remove it by Laparoscopy

C- give the medication

D- none

‫تكرر الموجات فوق الصوتية بعد الحيض‬

22) nurse would document the fetal station when the presenting
part in ischial spine .. it is?

A. +2

B. +1

C. 0

D.-1

23) 2 days post delivery mother has a bright color discharge lochia
and clots sized more than 1 cm, the funds were firm, What is the
proper intervention by nurse?

A- inform the doctor

B- document findings

12 ‫الصفحة‬ 0569945982 / ‫ محمد سامي جوال‬: ‫استاذ‬


24)24weeks pregnant women came to ER with vaginal bleeding and
hypertension, the nurse gives the priority to manage?

A-hypertension

B-bleeding

25)Which type of lochia the nurse expects to find in a client 2days


PP?

A-foul-smelling

B-lochia serosa

C-lochia alba

D-lochia rubra

26) Gestational diabetes glucose screening test performed at

A-6-12weeks

B-12-24weeks

C-24-28 weeks

D-36-48weeks

27)which of the following intervention would be helpful to a


breastfeeding mother who is experiencing engorgement breasts?

A-Applying rice

B-applying a breast binder

C-teaching how express her breast in a warm shower

13 ‫الصفحة‬ 0569945982 / ‫ محمد سامي جوال‬: ‫استاذ‬


D-administering bromocribtine

28)Which of the following matches the definition: abnormal


placenta development covering the cervix?

A- Placenta Previa

B-Abruptio Placentae

C-Multigravida

D- Proliferative phase

29)Which of the following terms correspond with the phrase: a


woman that is pregnant?

A- Gravida

B- Parity

C- Spermatogonia

D- Zona pellucida

30)Which of the following is the location where fertilization


occurs?

A- Ovaries

B- Vagina

C- Uterus

D- Fallopian Tubes

31) which of the following prevents pregnant avoid sleeping back?

A-Backache

B-Maternity tachycardia

C-O2 baby

14 ‫الصفحة‬ 0569945982 / ‫ محمد سامي جوال‬: ‫استاذ‬


D-Movement the baby

32)Before birth which is of the following structures connects the


right and left auricles of the heart?

A. Umbilical vein

B. Foramen ovale

C. Ductus ovale

D. Ductus venosu

33)A patient reports the onset of a vaginal discharge that is copious


foamy , foul smelling , and yellowish green colour . Which
organism is most likely causing the symptom?

A-Candida alibicans

B-Human papillomavirus

C-Treponeme pallidum

D-Trichomonas

34)A pregnant woman for the first time has no idea about
ovulation, As a nurse which the following is correct statement for
educating her?

A- happens about 14 days before your period start

35) Which of the following is definition of placenta abruption

a-is the separation of the placenta from the uterine lining

B-when the placenta attaches inside the uterus but near or over the
cervical opening
15 ‫الصفحة‬ 0569945982 / ‫ محمد سامي جوال‬: ‫استاذ‬
C-Fetal distress is an uncommon complication of labor. It typically
occurs when the fetus has not been receiving enough oxygen

D-none of the above

36)Patient arrives at the office today for her annual gynecological


appointment. She tells the nurse that she and her husband are
actively trying to get pregnant. What preconceptual counseling

With regard to vitamin supplementation, would the nurses give


her?

A-suggest she take a vitamins that contains 400 milligram of folic


acid

B-suggest she take a vitamins that contains 400 micrograms of folic


acid

C-suggest she take a vitamins that contains 2grams

of folic acid

D-suggest she take a vitamins that contains 4 grams of folic acid

37)What is Antepartum uterine fibroid treatment?

A-Hormone therapy

38)A patient who is gravida 8 present with missed periods for 7


weeks. P.V. bleeding. Cervical excitation, lower abdominal pain,
fainting 2 times yesterday. You diagnosis is

A-pancreatitis

B-ectopic pregnancy

C-abortion

16 ‫الصفحة‬ 0569945982 / ‫ محمد سامي جوال‬: ‫استاذ‬


39) baby delivered with high body weight compared with normal.
The reason could be is

A- diabetic mother

Nutrition

Diseases Type of required food

1-Celiac disease Gluten- free


2-Cholecysestitis High protein, high
carbohydrates, low fat
3-Cushing High k, low Na
4-Cystic fibrosis High calorie, high Na
5-Bed sore high protein
6-Mi Low fat, low NA, low
cholesterol
7-Nephrotic syndrome Low NA, normalprotein, high
calorie
8-Burn High calorie, high protein
9-Renal failure Low protein, high
carbohydrates , low NA
10-Hypothyroidism Low cholesterol, low calorie,
low saturated fat
11-Hyperthyroid High calorie, high protein

12-Ricketts Food contain VID such as Fish


, liver , milk, eggs ,Oatmeal

40)Which vitamin promote wound healing?

A-A

17 ‫الصفحة‬ 0569945982 / ‫ محمد سامي جوال‬: ‫استاذ‬


B-B

C-C

D-D

41) What is the proper diet After tonsillectomy

A-tomato’s salad

B-mashed potato’s soup

42) Depression and weakness related to a deficiency in which of the


following vitamins

A-Vitamin A

B-Vitamin b

C- Vitamin c

D-vitamin D

43)The nurse would instruct the client to eat which of the following
foods to obtain the best supply of vitamin B12?

A - Whole grains

B - Fish

C - Liver

D - Orange

44)you’re helping a mother, whose child was recently diagnosed


with Celiac Disease, read food labels. Which items below, if listed
as the ingredients, should the mother avoid feeding her child?

A-Millet

18 ‫الصفحة‬ 0569945982 / ‫ محمد سامي جوال‬: ‫استاذ‬


B-Wheat

C-Corn

D-Buckwheat

45) The nurse is preparing to care for a child after a tonsillectomy,


the nurse educate the parent about suitable type of food ..which of
the following should involve?

A- rice with meats

B- cheese macaroni

C- suchi

D-mashed potatoes and soup

46)Parkinson’s disease have a hard time swallowing because they


lose control of their mouth and throat muscles As a nurse which of
the following is a Parkinson’s diet?

A-Solid

B-Semi solid

C-Liquid

D-Clear liquid

47) Patient has cholecystitis the worst fat diet the nurse advice to
avoid is?

A-Whole milk

B- fish

48)which vitamins deficiency is most likely to be a long -term


consequence of a full- thickness burn injury?
19 ‫الصفحة‬ 0569945982 / ‫ محمد سامي جوال‬: ‫استاذ‬
A-vitamin A

B-vitamin B

C-vitamin C

D-vitamin D

49)Pt with higher cholesterol which diets can give for pt?

A-Meat

B-Banana

C-Rice

D-beans

50) The nurse is teaching a client who has Rickets about foods she
should include in the diet. The nurse determines that the client
under- stands the dietary modification if which items are selected
from the menu?

A-meat, rice

B-yogurt, milk

C-orang juice, tomatoes

51)which of the following helps to prevent neural tube defects in


pregnancy?

A-folic acid

B-B12

C-vitamin C

D-calcium

20 ‫الصفحة‬ 0569945982 / ‫ محمد سامي جوال‬: ‫استاذ‬


52) physician might recommend a Diet for patients with
Ventroperitum shunt,Which of the following?

A- peanut

B-Strawberry

C-Eggs

D-Latex

53 ) A child after spinal bifida surgery has complained of an


allergic reaction Which of the following could cause that?

A-Egg

B-Latex

C-Strawberry

D-Peanut Butter

54)Which of the following food is rich with vitamin D?

A-tomato

B-egg yolks

C-yogurt

D-Orange

55) iron supplemental should be taken with

A-orange juice

21 ‫الصفحة‬ 0569945982 / ‫ محمد سامي جوال‬: ‫استاذ‬


Calculations

56)A patient in the critical care area had a pre-injury body mass
index of 18. How would the nurse interpret this patient’s pre-injury
status?

A)As normal

B)As overweight

C)As underweight

D) As obese

57) Calculate the IV flow rate for 1000 mL of normal saline 0.9 to be
administered over 8hr The infusion set has drop factor of 10 gtts/m?

A-21drop /min

B-21ml/hr

C-125drop/min

D-125ml/hr

58) Patient with on going IV fluids 1400ml per24 hours and on


antibiotics 50mlQID and doctor order for fluids restrictions what is
the total IV fluids after fluid restriction

A-1200 ml per 24 hours

59) A baby 11 months has 7 times diarrhea how to calculate O.R.S


amount (700)?
22 ‫الصفحة‬ 0569945982 / ‫ محمد سامي جوال‬: ‫استاذ‬
A-7*100=700

60)Calculate the IV flow rate for 2000 mL of normal saline 0.9 to


be administered over 12h. The infusion set has drop factor of 15
gtts/m

A-42ml

B-42drop/min

C-64drop/min

D-none of the above

61)Doctor ordered 100mg Hydrocortisone And the available is


200mg in 5 ml Calculate how much you will give ?

A-2,5 ml

62)medication xDr order0.125 mg. Available: 0.25mgWhat is the


amount?

A-tow tablets

B-1 tablet

C-one half tablets

63) medication x Dr order 1500 with n/s 0,9 time12 how many ml
you will give in hour?

A-75ml/hr

23 ‫الصفحة‬ 0569945982 / ‫ محمد سامي جوال‬: ‫استاذ‬


B-125ml/hr

C-100ml/hr

D-200ml/hr

64) 8months child with vomiting and diarrhea since 3 days the
doctor order for oral hydration salt how many ml oral ORS per kg
you will give?

A. 25

B. 50

C 75

D. 100

65) the doctor prescribed 1000 gram adol tablet for patient. The
available dose is 250 mg What should the nurse do?

A- give 4 tablets to achieve the desired dose

B-inform doctor

C-call pharmacist

D-none

66)Which of the following considers hypotonic fluids?

A- NS 0,45

67) Order: Heparin 10000 units OD S/C Standard Solution 40000


units/ml How many ml would you administer?

A-0,25.ml

B-0,50

24 ‫الصفحة‬ 0569945982 / ‫ محمد سامي جوال‬: ‫استاذ‬


68)46 Xy chromosome is considerable?

a-normal in male

B-normal in female

C-abnormal in male

D-abnormal in female

Pediatric

69) A 6 year old male is diagnosed with nephrotic syndrome , in your


nursing care plan you will include What type of foods can the patient
eat?
A- high fat , high salt
B-low fat , high salt
C-normal protein, low salt
D-low salt ,low fiber

70)The nurse is providing care for a patient who has been admitted
to the hospital for the treatment of nephrotic syndrome. Which of
the following drug of choice for patient with nephrotic syndrome:-

A-prednisone(corticosteroids)

B-NSAIDs

C-ibuprofen

D-non of the above

25 ‫الصفحة‬ 0569945982 / ‫ محمد سامي جوال‬: ‫استاذ‬


71) A child with Jaundice and treat with photography ,how can
you protect the child skin?

A- Avoid oily substance

B-Using Vaseline after skin cleaning

C-applying cream

72) pt have Nephrotic syndrome which of the following color of


urine?

A-hematuria bright red

B-Yellow brown

C-orange

73) Which of the following pain tools assessment for 7 years child

A-Wong Baker faces pain

26 ‫الصفحة‬ 0569945982 / ‫ محمد سامي جوال‬: ‫استاذ‬


74) At the first minute baby crying, respiratory rate 44, acrosynsis,
muscle tone flaccid with manimum flextion and heart rate 120.
What is the total APGAR score?

75) During a prenatal visit, you are assessing the fundal height.
You find the fundus of the uterus At two third of the distance
between symphysis pubis and umbilicus . Based on this finding the
patient is about how far along in her pregnancy?

A-16

B-18

C-20

D-22

76)nurse is providing instruction to a mother gravida 8 Para 8 have


3 children diagnosed with dawn syndrome what she needs?

A-health class

77) At what age would you perform defintive lip repair following
lip adhesion?

A-3-6month

B-8-10 month

C-10-12 month

D->12 month

27 ‫الصفحة‬ 0569945982 / ‫ محمد سامي جوال‬: ‫استاذ‬


78)At what age would you perform defintive lip repair following lip
adhesion?

A-2month

B-1 year

C-2year

D-Non of the above

79)when performing nursing care plan for a neonate after a birth,


which intervention has the highest nursing priority neonate?

A-obtained a dextrostix

B-give the initial bath

C-Give the vitamin k injection

D-cover the neonate head with a cap

80)Which of the following is the most common site to obtain a


capillary blood sample from Neonates?

A-Heel

B-Earlobe

C-Fingertip

28 ‫الصفحة‬ 0569945982 / ‫ محمد سامي جوال‬: ‫استاذ‬


81)nurse administers albuterol nebulization to a child with asthma
exacerbation. The nurse Measures pulse oximetry and auscultates
the lungs to determine whether the goal of clear Respiratory status
has been met. The step of nursing is called?

A-Assessment

B-Diagnosis

C-Implementation

D-Evaluation

82)A child complained of diarrhea for 3 days 7 times / day What is


the severity of that?

A- mild

B- moderate

C- sever

83)An APGAR score for infant was 7 at the first minute then after
5 minutes the score became 10The child assessment is?

A- Healthy newborn

84) child came with hand structure and he was screaming What is
the first action of nurse?

A- Give him analgesic

B- Ask why you are screaming

C-calm the child


29 ‫الصفحة‬ 0569945982 / ‫ محمد سامي جوال‬: ‫استاذ‬
D- Abdomen

85) A child came with a hand fracture and when the doctor takes
the x-ray he found a previous fracture and healed What is nurse
action

A- Inform police

B- Ask the child how did that happen

C- Ask the family and give them trust to talk

D- Call the Doctor

86)A nurse is giving discharge planning instruction to the parents


of a 1-years old child with acute otitis media. Which of the
following discharge instruction take FIRST priority?

A-Administer antibiotics as prescribed

B-Breastfeed as long as possible

C-Administer influenza vaccination

D-Avoid smoking around the child

87) A 10 year old child has very limited vocabulary and interaction
skills. She has an I.Q. of 45. She is diagnosed to have Mental
retardation of this classification?

A. Profound

B. Mild

C. Moderate

D. Severe

30 ‫الصفحة‬ 0569945982 / ‫ محمد سامي جوال‬: ‫استاذ‬


88) A 8 years old child has a stubborn attitude and bad mood with
his family ,What is the nursing advice for that family?

A-its normal at his age he still has time to know the right

89)When conducting discharge teaching for the parent of a child


newly diagnosed with cystic fibrosis. Which of the following
statement by the parent indicates the need for further teaching?

A-Weekly weights help evaluate effectiveness of nutritional


interventions

B- Weekly weights help the doctor know if may child is absorbing


nutrients

C-Weekly weights reassure my child that recovery is progressing

D-Weekly weights help the doctor know if my child needs


additional enzymes

90)7year-old male patient is being evaluated for seizures. While in


the child’s room talking with the child’s parents, you notice that

31 ‫الصفحة‬ 0569945982 / ‫ محمد سامي جوال‬: ‫استاذ‬


the child appears to be daydreaming. You time this event to be 10
seconds. After 10 seconds, the child appropriately responds and
doesn’t recall the event. This is known as what type of seizure?

A-Focal Impaired Awareness (complex partial)

B-Atonic

C-Tonic-clonic

D-Absence

91)A child with asthma is experiencing thick respiratory secretions


Resulting in increased work of breathing. The best nursing
Intervention is to:

A-Encourage fluids

B- Eliminate dairy products

C-Decrease relative humidity of the room

D-Have the child lay on the left side

92)The nurse is preparing to care for a child with a diagnosis of


intussusception. The nurse reviews the child’s record and expects
to note which sign of this disorder documented?

A-watery diarrhea

B-ribbone-like stools

C-profuse projectile vomiting

D-Red jelly stool


32 ‫الصفحة‬ 0569945982 / ‫ محمد سامي جوال‬: ‫استاذ‬
93)A public health nurse was giving genetic counselling to a couple
with sickle cell anemia. The Nurse assessed on the chances of their
children inheriting disease. What is the chance of their Each child
inheriting the disease?

A-25%

B-50%

C-75% )

D-100%

94)for an infant with hydrocephalus, a nurse should plan to


monitor for what sign or symptom of increased intracranial
pressure?

A-High- pitched cry

B-decrease in systolic blood pressure

C-depressed fontanels

D-increase in respiration

95) which of the following assessments findings would the nurse


most expect to find in the Child who has been diagnosed with
having hyperactive pyloric stenosis?

A-current jelly stools and palpable, hard mass in the right upper
quadrant

B-projectile vomiting and hunger soon afterwards

33 ‫الصفحة‬ 0569945982 / ‫ محمد سامي جوال‬: ‫استاذ‬


C-weight loss and bloody diarrhea

D-sever crampy abdominal pain and lethargy

96)A baby during feeding has a clear secretions and coughing As a


nurse which the following is correct statement for educating her?

A- Stop feeding and start suction

97)Baby born in 28 weeks his Wight is 850 gm, immediately he


transferred to NICU to complete his assessment, What should the
nurse document?

A- weight lower than gestational age

B- weight is appropriate to gestational age

C- weight very lower than gestational age

D- none

98)Diabetic mother delivered in the 38 weeks , the baby transferred


to NICU, what do you expect to happen to newborn?

A-hypoglycemia

34 ‫الصفحة‬ 0569945982 / ‫ محمد سامي جوال‬: ‫استاذ‬


99)a mother and 7 month old infant present to the pediatric
clinic.The infant appears developmentally appropriate and
healthy, but the mother tells you that she is exacerbated. She says
yesterday her infant had been incessantly crying with vomiting
And jelly like stool. But now the infant appears fine, which of the
following GI disorders does the nurse suspect?

A-hypertrophic pyloric stenosis

B-Celica’s disease

C-intussusception

D-encopresis

100)the infant with cleft Lip/palate needs more frequent of which


of the following?

A-burping after ever 15-30 ml

101)describes the nipples on bottles used to feed babies with cleft


lip

A-large holed, soft nipples

102)neonate of mothers with diabetes are at risk for complications


following birth?

A-atelectasis

B-macrosomia

35 ‫الصفحة‬ 0569945982 / ‫ محمد سامي جوال‬: ‫استاذ‬


C-microcephaly

D-pneumothorax

103) The nurse taking care of a woman who has gestational


diabetes and is in labor knows the patient and fetus are at risk for?

A-shoulder dystocia

B-late declarations

C-hypoglycemia

D-All of the above

Vaccination
B-hyper glycaemia

104)During a routine pediatric visit, a 18 month old patient will


need which of the following vaccines?

A-Hepatitis A

B- hepatitis B

C- measles

D-BCG

105)What type of vaccine for a baby 6 months?

36 ‫الصفحة‬ 0569945982 / ‫ محمد سامي جوال‬: ‫استاذ‬


A--DPT

106) a 1-year child came to the clinic to receive MMR vaccine


What does MMR mean?

A- meningitis, mumps, rubella

B- measles, mumps, rubella

C- measles, meningitis, Rota

D- none

107)Which vaccine is given soon after birth?

A-Polio

B-Hepatitis B

C-Measles, mumps, and rubella (MMR)

D-Varicella

108)Bacterial meningitis strikes infants more often than any other


age group. Which vaccine Will help prevent one previously
common type of meningitis?

A-Tetanus

B-Hib

C-HIv

D-Varicella

109)What is the vaccine which giving before al Hajj season?

37 ‫الصفحة‬ 0569945982 / ‫ محمد سامي جوال‬: ‫استاذ‬


Meningitis and flu,

110)A 2 month old is scheduled for routine immunizations. As the


nurse you know the physician will most likely order what
vaccinations?

A- Hepatitis A

B- Hepatitis B

C- Measles

D- Rubella

111)A 6 month old is scheduled for routine immunizations. As the


nurse you know the physician will most likely order what
vaccinations?

A-Hepatitis A

B-Hepatitis B

C- Measles

D- Rubella

112)A 9 month old is scheduled for routine immunizations. As the


nurse you know the physician will most likely order what
vaccinations?

A-Hepatitis A

B- Opv

C- Measles

D- Bcg

38 ‫الصفحة‬ 0569945982 / ‫ محمد سامي جوال‬: ‫استاذ‬


113)A 12 month old is scheduled for routine immunizations. As
the nurse you know the physician will most likely order what
vaccinations?

A- Hepatitis A

B- non of the above

C- MMR

D- BCG

114)A 18 month old is scheduled for routine immunizations. As the


nurse you know the physician will most likely order what
vaccinations?

A- Hepatitis A

B- Opv

C- MMR

D- Bcg

Infection control

Type of isolation :

- 1-Airborne 2-Droplet 3-Contact 4-Universal/standard


precaution

Airborne Droplet Contact Universal/


precautions precautions precations standard
39 ‫الصفحة‬ 0569945982 / ‫ محمد سامي جوال‬: ‫استاذ‬
TP(Tuberculosis) Mumps MRSA AIDS
Small box Pneumonia Scabies Hepatitis
A-B-C
Varicella Rubella Herpes Anthrax
(chicken box)
Measles Meningitis RSV Lyme
disease
Diphtheria Staphylococcal
Ebola

115)In order to reduce the risk of disease trnasmission from a


patient with diphtheria, which following standard precautions
would be the nurse implemented?

A.droplets

B.airborne

C.ventilatory

D.contact

116)A patient a demitted a virulent bacterial infection, which


causes risk infection through physical contact Which of the
following infection control precautions would the nurses use to
minimize the risk of the infection for this organism?

A-Droplet

B-contact

C-Airborne

40 ‫الصفحة‬ 0569945982 / ‫ محمد سامي جوال‬: ‫استاذ‬


D-transmission

117) What is the most effective procedure to prevent spread


infection?

A- hand washing

118) Which of the following is the correct order for putting on


personal protective equipment?

A- gown, mask, goggles,gloves

B- mask, gown, goggles, gloves

C- goggles, mask, gloves, gown

D- gloves, gown , goggles, mask

119)A patient with measles what should the nurse wear before
entering the patient room?

A-gloves and mask

B-gloves, gown and N95

C-gown and gloves

D-Mask and gown

120)in what order would you doff(remove) the PPE?

A-Gloves the mask

B-Goggles then mask

C-Gloves, goggles, gown, then mask

D-gloves, mask, Google, the gown

41 ‫الصفحة‬ 0569945982 / ‫ محمد سامي جوال‬: ‫استاذ‬


121)women in labor how the nurse can avoid her from infection?

A-use clean technique

B-avoid vaginal examination

C-maintaine warm room environment

122)A patient diagnosed with TB and transfer him to the isolation


room Who should wear the mask?

A- The medical staff

B- His family

C-Anyone who entered the patient room

D- Anyone who contact with patient

123)a client found to have tuberculosis (TB) is scheduled to go to


the radiology department for a chest radiography. Which one
should aware mask?

A-Nurse

B-Patient

C-Nurse and patient

D-Radiology technician

42 ‫الصفحة‬ 0569945982 / ‫ محمد سامي جوال‬: ‫استاذ‬


Leaders and administration

124)A newly nursing director assigned to a hospital What is the


first action should he do?

A-Evaluates the staff

B-change the head nurses

C-change the roles

43 ‫الصفحة‬ 0569945982 / ‫ محمد سامي جوال‬: ‫استاذ‬


D-nothing to do

125) A patient after umbilical hernia surgery lost her staff While the
nurse looking for that she found it in the cleaner bag .what is the
nurse action?

Incident report

126) Shortage in staffing and problem happened, charge nurse


should inform the

A-Nursing supervisor

127)Code blue for arrested patient the physician insists to perform


a CPR and be responsible about patient, What style of leadership
describes the physician attitude?

A-Orientation staff

B-Infection control

C-Safety team

D-Head nurse

128)a mother applies a nonmedical substance on her child burn,


the burn after that become worst when a nurse advised her to
avoid that What is that role?

A-Advocacy

B-Responsibility

44 ‫الصفحة‬ 0569945982 / ‫ محمد سامي جوال‬: ‫استاذ‬


129) A manger delighted some tasks to staff What is the first action
should the staff do?

A-read the policy of delegation

B-let the charge do some

130) A manger Nurse tell one staff that I will evaluate you in next
month these action is

A-feedback

B-assignment

131)the head nurse of your department has a democratic style,


what does that mean?

A-(all the nurses share and participate in decision)

132)the head nurse explain the meaning of codes?

A-implementatio

133)If the head nurse assigned the staff What is that action called?

A- delegation

134)A nurse gives patient a health education,Which step of the


nursing process could be that?

A-implementation

45 ‫الصفحة‬ 0569945982 / ‫ محمد سامي جوال‬: ‫استاذ‬


135)If you are a nursing director in a hospital, after meeting with
staff, What you should say at the end of meetings?

A- i am going to follow up you feedback monthly

136)a type of leadership style in which members of the group take


a more participative role in the decision-making process. This type
of leadership can apply to any organization, from private
businesses to schools to government. Which of following the
leadership style?

A-Bureaucratic

B- Democratic

C- autocratic

D-laissez fair

137)A supervisor delegates some tasks for the staff which of the
following steps could be?

A- Delegation

46 ‫الصفحة‬ 0569945982 / ‫ محمد سامي جوال‬: ‫استاذ‬


138) nurse is completing the preoperative checklist for one of the
patients who ring is wearing a ring.What is the most appropriate
action?

A. Give ring to security office

B. Lock ring with patient's valuables

C. Call patient's family to give them the ring

D. Respect patient's choice and leave ring on patient's finge

139)Patient was putting her gold ring under pillow late she lost,
when the nurse looked for it she found it in cleaner bag then the
nurse wrote an incident report Why the nurse wrote that?

A-follow the hospital policy

140)Patient was putting her gold ring under pillow late she lost

When the nurse looked for it she found it in cleaner bag, what is the

Nurse action?

Incident report

47 ‫الصفحة‬ 0569945982 / ‫ محمد سامي جوال‬: ‫استاذ‬


141) A co-worker informs the nurse about experiencing increased
level of stress associated with daily job responsibilities. To help cope
with professional stress, what should the nurse encourage?

A. Make a list daily of unfinished tasks

B. Complete complex mental tasks before physical tasks

C. Acknowledge daily accomplishments

D. Spend time with colleagues away from work

48 ‫الصفحة‬ 0569945982 / ‫ محمد سامي جوال‬: ‫استاذ‬


Psychiatric

142)A 36 year-old woman was admitted to the Psychiatric Unit


following an acute psychotic episode in which she was having
auditory hallucinations. She has a flat effect and she jumps from
one topic to another. Her sister questions if she has been taking her
medications regularly as prescribed. Which clinical finding would
most confirm that she has not been taking her medications?

A-Lack of arm swing while walks

B-Absence of emotional expression

C-Physical slowness of movement

D-Forward and backward moving of jaw

143)Psychiatric patient ,while the nurse try to give him medication


the patient refuse and said here not my place , aim trump the
resident and supposed to be in USA. Which of the following type of
delusion for this patient?

A-delusions of persecution

B-delusions of influence

C- delusions of reference

D-delusions of grandeur

144)Which of the following type of phobia for this patient

‫الخوف من المرتفعات العاليه‬A-Acrophobia

49 ‫الصفحة‬ 0569945982 / ‫ محمد سامي جوال‬: ‫استاذ‬


145) Which of the following type of phobia for patient with fear of
dogs?

A-cynophobia

‫الخوف من الكالب‬

146)The nursing staff in the female psychiatric Ward are scheduled


foe their lunch breaks . the medication nurse decides to administer
the due medication to the remaining two patients after returning
from the break . the patient are diagnosed as having manic
depressive psychosis . The accompanying staff is also aware of her
decision but none of the two nurses informed the charge nurse
regarding the decision . Breach in performing the necessary duties
is evaluated in which of the following domains?

A- Criminal offense

B- Legal accountability

C- Patient bill of rights

D- Health care system regulations

50 ‫الصفحة‬ 0569945982 / ‫ محمد سامي جوال‬: ‫استاذ‬


147)A 26-year-old woman is newly diagnosed with bipolar
disorder and has been prescribed lithium carbonate 300
milligram, four times daily, one week after beginning the
medication, she telephones the consulting nurse at the
psychiatric clinic with complaints off diarrhea and blurred
vision that had begun one day previously. The nurse schedules
her come to the office to see the doctor that day Which hormone
imbalance is most likely?

A-Estrogen

B- Luteinizing

C- Aldosterone

D-Thyroid-stimulating

148)A client comes to the outpatient psychiatric clinic for the


treatment of a fear of high building. The nurse knows that
phobias involve which of the following?

A-projection and displacement

B-sublimation and internalization

C-rationalization and intellectualiaztion

D-reaction formation and symbolization

51 ‫الصفحة‬ 0569945982 / ‫ محمد سامي جوال‬: ‫استاذ‬


Fundamental and adult

149)during assignments on surgical floor which of the following


tasks might be delighted to new nurse ?

A-discharge patient and explains medication

B-observes patients after post operation

C-caring bed rest patients

D-change patients position every 2 hours

150)A client is admitted with a diagnosis of acute appendicitis.


When assessing the abdomen, the nurse would expect to find
rebound tenderness at which location?

a) Left lower quadrant

b) Left upper quadrant

c) Right upper quadrant

d) Right lower quadrant

151) Wilma’s tumor should be removed quickly and avoid

A- don’t palpate abdomen

152)Dr. Mohammad Sami has determined that the client with


hepatitis has contracted the infection form contaminated food. The
nurse understands that this client is most likely experiencing what
type of hepatitis?

A-Hepatitis A

B-Hepatitis B
52 ‫الصفحة‬ 0569945982 / ‫ محمد سامي جوال‬: ‫استاذ‬
C-Hepatitis C

D-Hepatitis D

153) Hirschsprung

constepation and Falid thrive-

154)To prevent prolapse

A- Kegl’s exersic

155)when the shiny portion of the placenta comes out first, this is
called the mechanism.

A-Schultz

B-ritgens

C-Duncan

D-marmets

156)which of the following would the nurse most likely to expect to


find when assessing a pregnant client with abruption placenta?
A- excessive Vaginal bleeding
B-rigid board abdomin
C- titanic uterine contractions
D- premature rupture of membranes

157) A newly RN nurse is about to remove a nasogastric tube to a


client with Guillan Barre Syndrome. To determine the proper
Time of removing the tube, the nurse should do it when

A-when the client feeling hungry

B-when the client want to remove it

C-when the client totally conscious

53 ‫الصفحة‬ 0569945982 / ‫ محمد سامي جوال‬: ‫استاذ‬


D- none of the above

158) Which pulse should the nurse palpate during “ rapid


assessment of an “ unconscious male adult ?

A - Radial

B - Brachial

C - Femoral

D – Carotid

159) Watery diarrhea like rice is most often caused by what type of
bacterium?

A-pneumococcus

B-streptococcus

C-cholera

D-klebsiela

160) What is the medication that can be given for bradycardia?

A- atropine

161) What is the instruction about taking omiprazol

A- tablet before meal 30 to 60 minutes

162) A patient after checking ABO and starting blood transfusions


he has rash, itching.. what should the nurse do

A- stop infusion

54 ‫الصفحة‬ 0569945982 / ‫ محمد سامي جوال‬: ‫استاذ‬


163) What is the contraindication of oral contraceptive biles

A- thrombophlebitis

164) What is the action of nitroglycerin?

A- increase blood supply to the heart

165) Common causative of diarrhea?

A- shigella

B- bacillus

C- Rota

166) A nurse is giving im injection for a patient with HIV and


suddenly puncture his finger, what should the nurse do?

A- Do test immediately

B- Do test after 6 months

167) A patient with COPD & CHF, what should be done to the
patient?

A- CKMB analyst

55 ‫الصفحة‬ 0569945982 / ‫ محمد سامي جوال‬: ‫استاذ‬


168) see image below is :

A-Mongolian spot

56 ‫الصفحة‬ 0569945982 / ‫ محمد سامي جوال‬: ‫استاذ‬


169) Site of gastroesophageal reflux disease?

a-A

b-B

c-C

d-non

170) a nurse in male surgical word assigned to different tasks all at


the same time. The nurse managing 4 patients all with different
needs and concerns of varying urgency, which of the following
considering critical and need the priority of care…

A- patient diabetic with right leg pressure ulcer

B-patient with left diabetic foot

C-patient after appendectomy

D-patient has dressing for a head injury

171) patient has burn in left foot, the patient underwent surgery
for skin grafting after the surgery the Nurse observed blood spots
on the dressing What should the nurse perform?

A-pressure over the dressing

B-put a circle with a pen around the blood and check later

C-inform doctor

D-none

57 ‫الصفحة‬ 0569945982 / ‫ محمد سامي جوال‬: ‫استاذ‬


172)A client with cystic fibrosis is taking pancreatic enzymes. The
nurse should administer this medication:

A-Once per day in the morning

B-Three times per day with meals

C-Once per day at bedtime

D-Four times per day

173) A 23 year- old male comes to the Emergency Department in a


sickle cell crisis. He reports that his pain level is a 10110 in all
extremities. During the assessment, he cannot lie still because of the
pain. There is no cyanosis or clubbing in the extremities and all
examination findings are normal. The vital signs recorded
were82/132 mmHg Blood pressure110/min Heart rate18/mm
Respiratoryrate°38.4C Temperature 94 an % room air Oxygen
Saturation Which nursing diagnosis is first priority?

A - acute pain

B - fluid volume deficit

C - ineffective tissue perfusion

58 ‫الصفحة‬ 0569945982 / ‫ محمد سامي جوال‬: ‫استاذ‬


D - ineffective airway clearance

174) The nurse should visit which of the following clients first?

A-The client with diabetes with a blood glucose of 95mg/dL

B-The client with hypertension being maintained on Lisinopril

C-The client with chest pain and a history of angina

D-The client with Raynaud’s disease

175)A nurse has broken a medication vial by mistake What the


first action should the nurse do?

A-inform the head nurse

B-change the vial from the pharmacy

C-Write a report

D-Documenting in nursing note

176)You are just assigned as nursing director,What is the first


action performing to improve your department?

A-Check what are the last 3 achievements in the department

B-apply new assertive roles

59 ‫الصفحة‬ 0569945982 / ‫ محمد سامي جوال‬: ‫استاذ‬


C-meeting the staff individually and ask them about needs

D-None

177)An old patient came to the ER with bruises and fracture and
this consider the fourth time in this year with same complains As a
nurse what is your action?

A-Inform the police with report


B-Inform doctor
C-Ignore that
D-None
178)Patient came to his a scheduled appointment for follow up
when the nurse check the vital signs BP: 130/92 RR: 18 HR: 86T:
37Which of the vital signs is abnormal?

A- Temperature
B- Heart rate
C- Respiratory rate
D- Blood pressure
179)Patients came to the ER complaint of vomiting and diarrhea,
she was lethargic When the nurse asks her about first name and
place she answered the appropriate and she knows she is in a
hospital What is the proper place to check her temperature?
60 ‫الصفحة‬ 0569945982 / ‫ محمد سامي جوال‬: ‫استاذ‬
A- Axillary
B- Orally
C- Tympanic
D- rectal

180)A school nurse was explaining for 14 years old students and
suddenly one of them said there is a small age gab between us so
don’t pretend to be more understanding than me What is the
appropriate answer should the nurse said?

A-I am older than you

B-Let’s focus se on the topic

C-You are able to explain better than me

181) The rectal temperature, a core temperature, is considered to


be one of the most accurate routes. How many centimeters should
the nurse inserts the thermometer through the rectal ?

A-1,5 cm

B- 2 cm

C- 2,5 cm

D-1 cm
61 ‫الصفحة‬ 0569945982 / ‫ محمد سامي جوال‬: ‫استاذ‬
182)The nurse cares for a school age client with congestive heart
failure. Which clinical finding indicates adherence to the
prescribed medical regimen?

A- No change in daily weight

B- Absence of nausea or vomiting

C- Respiratory rate 34 breaths/min

D-Pulse rate 120 beats/min

183)The diagnosis of flat facial feature, small head , short ear


,short neck poor muscle tone?

A- dawn syndrome

B- Gillian berry syndrome

C- autism spectrum

D- none

184) Patient exhibits no response to loud voice. according to


Ramsey score sedation scale ( RSS) which score could give to that
patient?

A-1

B-2

C-5

62 ‫الصفحة‬ 0569945982 / ‫ محمد سامي جوال‬: ‫استاذ‬


D-6

185)What is the method of giving nitroglycerin treatment?

A-undertounge

B-intradermal

C-oral spray

D-non if the above

186) If the nurse wants to teach the patient about hand hygiene
then using posters in a hospital’s corridor What would the nurse
do ?

A- planning

B- assessment

C- implementation

D- evaluation

187) A mother told the nurse that ( I love my kids and I wouldn't
want more )Which of the following describe the mother behavior?

A- denial

B- anger

C-acceptance

63 ‫الصفحة‬ 0569945982 / ‫ محمد سامي جوال‬: ‫استاذ‬


D- bargaining

188) As a nurse why should we perform a review of the discharging


plan ?

A- to check the investigation

B- to observe the progress of discharge process

C- for health education

D-none

189)A nurse is preparing to care for a client who has returned to


the nursing unit following cardiac catheterization performed
through the femoral artery. The nurse checks the physician’s
prescription and plans to allow which client position or activity
following the procedure:

A-Bedrest in high Fowler’s position

B-Bedrest with bathroom privileges only

C-Bedrest with head elevation 60degrees

D-bedrest with head elevation no greater than 30 degrees

190)the nurse is caring for a client with an above – the knee-


amputation (AKA). To prevent contracture, the nurse should

A-place the client in a prone position 15-30 minutes twice a day


64 ‫الصفحة‬ 0569945982 / ‫ محمد سامي جوال‬: ‫استاذ‬
B-keep the foot of the bed elevated on shock blocks

C-place trochanter rolls on either side of the affected leg

D-keep the client’s leg elevated on two pillows

191) The doctor prescribed lithium for A bipolar patient when the
nurse check the serology, the sodium was low. Relating to the result
the doctor ordered urin analysis to examine which of the following

A- concentration

B- osmolality

C- filtration

D- none

192) a nurse in labor room is performing a vaginal assignments on a


pregnant client in labor. The nurse notes the presence of the
umbilical cord protruding from the Vagina. Which of the following
would be the initial nursing action?

A-place the client in Trendelenburg ‘position

B-Call the delivery room to notify the staff that the client will be
transported immediately

C-Gently push the cord into the Vagina

193) The Nurse received a patient who is coming from the ER to the
surgical ward Which action could be the priority of the nurse to do

A- document about the case

B- take the intake and output e

65 ‫الصفحة‬ 0569945982 / ‫ محمد سامي جوال‬: ‫استاذ‬


C- check the vital signs

D- assess the incision site

194)A nurse is assessing a client with possible cushion’s syndrome,


the nurse expects to find:

A-weight gain in arms and legs

B- thick, coars skin

C-hypotension

D-deposits of adipose tissue in the trunk and dorcerervical area

195)when performing a newborn assessment, the nurse should


measure the vital signs in the following sequence:

A-pulse, respiration, temperature

B-temperature, pulse, respiratton

C-respiration, temperature, pulse

D-respiration, pulse, temperature

196)what disease is associated with a vitamin C deficiency?

A-Pellergia

B-nural tube defects

C-scurvy

66 ‫الصفحة‬ 0569945982 / ‫ محمد سامي جوال‬: ‫استاذ‬


D-pitted edema

197)when caring for a patient with left- sided homonymous


hemianopsia resulting from a stroke, which intervention should the
nurse include in the plan of care during acute period of the stroke?

A-Apply an eye patch to the left eye

B-approach the patient from the left side

C-place objects needed for activities of daily living on the patient’s


right side

D-reassure the patient that the visual deficit will resolve as the stroke
progresses

198)a patient receiving a total parenteral nutrition through a central


line suddenly has difficulty breathing and is restless. Chest
auscultation reveals a heart murmur.

Blood pressure 90/60mmhg

Heart rate. 120/min

Respiratory rate. 22/min

Temperature. 37.1C

Oxygen saturation. 90%on room air

What is the most appropriate initial nursing action?

A-notify the physician

B-administer 100% oxygen by face mask


67 ‫الصفحة‬ 0569945982 / ‫ محمد سامي جوال‬: ‫استاذ‬
C-place patient in lift side lateral decubitus

D-obtain Stat blood glucose measurement

199) the nurse is caring for a patient receiving IV furosemide (Lasix)


40mg and enalapril(Vasotec) 5mg PO bid for ADHF with severe
orthopnea. When evaluating the patient response to the medication,
the best indicator that the treatment has been effective is:

A-weight loss of 2 pounds overnight

B-improvement in hourly urinary output

C-reduction in systolic bp

D-decreased dyspnea with a head of the bed at 30 degrees

200)a client is unresponsive and has been brought to the Emergency


Department. Initial laboratory results reveal

Serums k+. 3.6mmol/L

Glucose. 26mg/dl

Hemoglobin 12.6gm/dl

Carbon dioxide. 26. 2mmol/L. The nurse will anticipate:

A-Dextrose by mouth

B-50%dextrose IV

C-1unit of packed cells

68 ‫الصفحة‬ 0569945982 / ‫ محمد سامي جوال‬: ‫استاذ‬


D-10mEq of KCL over 0.5hour

201) A doctor was exhausted after work, he asked the nurse to inject
him with morphine

What could the nurse do before giving?

A- the nurse refused

B- give him the injection

C- ask another doctor to manage him

D- ask the doctor to write the prescription

202) Patient visiting the clinic 10 days after sinus surgery for checkup
complains of having a bad taste in the mouth. When the nurse smells
a foul odor while examining the patients mouth, the nurse suspects
the patient have an:

a. Pulmonary decompensation

b. Hemorrhage

c. Aspiration

d. Infection

69 ‫الصفحة‬ 0569945982 / ‫ محمد سامي جوال‬: ‫استاذ‬


203)The community midwife has just finished an education session
with a group of women regarding maternal health and annual
gynecological examination along with mammogram and Pap smear
testing.

What outcome is the most desired?

A-Disease prevention

B-Increased healthy living

C- Early detection of related issues

D-Strengthened marital relationship

204)A 69-year-old female patient in the Medical Ward is in a semi-


conscious state. Her pancreatic cancer is metastasized to her liver
and lungs and she is admitted for supportive treatment. Her
physicians discussed with her family that she will not be given the
CPR to save her life if she goes into the cardiac arrest. Her two sons
agreed but the daughter is indecisive. What is the critical thinking
behind not providing cardiopulmonary resuscitation?

A-Unilateral judgment of health professionals

B-Refusal of patients right to treatment

C-Ethical dilemma and is indecisiveness


70 ‫الصفحة‬ 0569945982 / ‫ محمد سامي جوال‬: ‫استاذ‬
D-Mercy killing to ease sufferin

205) Which of the following best describes the role of a case


management?

A-Promote healthy lifestyle

B-Provides coordination of care to ensure continuity

C-Possesses highly skilled communication methods

D-Promotes the comfort of the patient

206)complication of gastrectomy and peptic ulcer is:

A. Hemolytic anemia

B. Pernicious anemia

C. Thrombocytopenia

207) the nurse begin a morning shift with the following patients
waiting to see the doctor in the waiting room

1-A first time mother shift delivered two days ago and is having
difficulties breastfeeding
2-a 60 years – Old man who has a history of deep vein thrombosis
and taking daily heparin
3-A76 years – old women who has hosppitalzed three days
previously with pneumonia
4-A 56years – old man who has used all his diuretics and is coughing
up sputum

Which patient should be seen first :

A-one
71 ‫الصفحة‬ 0569945982 / ‫ محمد سامي جوال‬: ‫استاذ‬
B-two

C-three

D-four

208)A nurse called the doctor while he is at home when she asked
him for an order for patients medication As a nurse how many hours
must the doctor took to write the order in the hospital
documentation?

A- 8 hours
B- 24 hour
C- 48 hours
D- 12 hours

209) The maximum time for suctioning in adults is:

A‐ 5 second

B‐ 8 seconds

C‐ 10 seconds

D‐15 second

210)When involved in prenatal teaching. The nurse should advise the


clients that an increase In vaginal secretions during pregnancy is
called leukorrhea and is caused by increased?

A-metabolic rates

B-production of estrogen

72 ‫الصفحة‬ 0569945982 / ‫ محمد سامي جوال‬: ‫استاذ‬


C-functioning of the bartholin glands

D-supply of sodium chloride to the cells of Vagina

211) Which of these is not considered a T.O.R.C.H.infection?

A- Rubella

B- Herpes

C- Cytomegalovirus

D-croup

212) Where does spermatogenesis occur?

A- Seminiferous tubules

B- Corpus spongiosoma

C- Prostate gland

D-scrotum

213) A 50 year-old man present to the Emergency Department after


having suffered a fall while practicing mountain biking. he appears
confused and disoriented . his friend reports that he had been thrown
over the handle bars and hit his head against a rock

BP 108/86

HR 102/min

RR 22/min

73 ‫الصفحة‬ 0569945982 / ‫ محمد سامي جوال‬: ‫استاذ‬


Tem 37.2

O2 SAT 99% Which initial intervention is the most important?

A. Trendelenburg position

B. Intravenous infusion

C. Oxygen administration

D. Cervical immobilisation

214) a student nurse is learning to assess BP. what does the BP


measure?

A. force of blood against atrial walls

B. flow of blood through the circulation

C. flow of blood through the heart

D. force of blood against venous walls

215)A 32 year-old woman being hospitalized since six days for


obesity treatment she reported to the nurse “ I feel burning in my left
calf for the last two days “ after nursing assessment the nurse found
that a left thigh oedema , leL thigh circumference is more by 8 cm
than right thigh and it is warmer to touch ,no trauma or wounds.
What is the most possible cause for that?

A. Obesity

B. Local cellulitis
74 ‫الصفحة‬ 0569945982 / ‫ محمد سامي جوال‬: ‫استاذ‬
C. Hypertension

D. Deep vein thrombosis

216) )A 19 year-old patient is admitted to the Male Surgical Word


after the road traffic accident in the car racing with his friends. He
had fractured bones in his left sided extremities including fibula in
his leg. His wrist. The cast is applied on his arm as well as on his leg.
His physiotherapy sessions will be started according to the evaluation
of his treating surgeon. What short term goal should be planned
while considering the age of the patient?

A. Personal counselling for safe driving practice

B. Importance of body image for the patient

C. Respect patient privacy needs

D. Encourage acavity and exercise

217) A 4 years boy came to ER with fracture in the right fibula and
tibia. When x-raw wad done to him, it showed that he had same
fracture in the same position previously . What should the nurse do?

A-ask the boy about the incidance

B-call the social worker

C-call police

D-call supervisor

75 ‫الصفحة‬ 0569945982 / ‫ محمد سامي جوال‬: ‫استاذ‬


218)A patient has a chest tube, the nurse transferred him to
radiology for x-ray on a stretcher. Where should the nurse put the
chest tube bag;

A-hang on iv stand

B-put it beside the patient

C-hang it under the stretcher

219)a cardiac monitor for a patient in a coronary abnormal ECG


rhythm with a heart rate of 159 be a complex(0.18 second), and
absent p wave.What could be the type of possible dysthymia?

A-sinus tachycardia

B-ventricular tachycardia

C-ventricular fibrillation

D-suprqventricular tachycardia

220) A40year-old man is admitted to the hospital following a motor


vehicle accident in which he received a head injury . on admission ,
the patient is alert active and oriented . He is on observation for
potentially increasing intracranial pressure . Which clinical findings
should the nurse initially monitor most carefull?

A. Decreased pulse and increased blood pressure

76 ‫الصفحة‬ 0569945982 / ‫ محمد سامي جوال‬: ‫استاذ‬


B. Irregular respiration patterns or cheyne-satokes

C. Bilaterally dilated and no-reactive pupils

D. Changes in level of consciousness

221). A surgical nurse is assigned to care for a woman who had


undergone vaginal hysterectomy. The nurse performed assessment .

Which of the following potential problem that the nurse should


observe?

A. Pain

B. Fever

C. Anxiety

D. Bladder dysfunction

222) A nurse is caring for a 30year-old comatose patient who was


admitted with subdural hematoma. Intravenous mannitol was given
to the patient. What the nurse should notice if mannitol was used for
a long term ?

A. Hyperthermia

77 ‫الصفحة‬ 0569945982 / ‫ محمد سامي جوال‬: ‫استاذ‬


B. Puplis non-reactive

C. Urine output decrease

D. Increased blood pressure

223)A nurse is assessing a patient who of becoming short of breath


easily intolerant of activity and has a productive cough. The nurse
notices increased anteroposterior diameter (barrel chest). The patient
smokes two packs of cigarettes a day. Which of the following
diagnosis is most likely ?

A. Cor pulmonale

B. Lung abscess

C. Emphysema

D. Chronic bronchitis

224)A 32 year-old woman being hospitalized since six days for


obesity treatment she reported to the nurse “ I feel burning in my left
calf for the last two days “ after nursing assessment the nurse found
that a left thigh oedema , leL thigh circumference is more by 8 cm
than right thigh and it is warmer to touch ,no trauma or wounds.
What is the most possible cause for that?
78 ‫الصفحة‬ 0569945982 / ‫ محمد سامي جوال‬: ‫استاذ‬
A. Obesity

B. Local cellulitis

C. Hypertension

D. Deepveinthrombosis

225) A community health nurse screens a group of high-risk adults


for tuberculosis Which gauge needle should the nurse use for an
intradermal injection on the volar surface of the forearm?

A-16 gauge needle

B-20 gauge needle

C-22 gauge needle

D-26 gauge needle

226) Night work causes lack of any of the following vitamins?

.A-vitamin A

B-vitamin B

C-vitamin c

79 ‫الصفحة‬ 0569945982 / ‫ محمد سامي جوال‬: ‫استاذ‬


D-vitamin d

227)Sign of hyperthyroidism

Exophthalmos

228) patient with a brain injury resulting in a cerebral edema. Your


ability to recognize cerebral edema early and intervene appropriately
can improve patient outcomes , the Nurse should be able to recognize
signs, symptoms and the medication which can reduce the ICP(
intracranial pressure) which is

A-lasix

B-atropin

C-digoxin

D-mannitol

229) Randy has undergone kidney transplant, what assessment would


prompt Nurse Katrina to suspect organ rejection?

A-Sudden weight loss

B-Polyuria

C-Hypertension

D-Shock

230) the nurse evaluating nursing care plan for 50 years old female
who has been scheduled for lithotripsydue to urinary calculus which
of the following is the priority evaluation?

A-edema

B- acute pain

80 ‫الصفحة‬ 0569945982 / ‫ محمد سامي جوال‬: ‫استاذ‬


C-knowledge of deficit

D-UTI

231)A conscious victim of a motor vehicle accident arrives at the


emergency department. The patient is Gasping for air, is extremely
anxious, and has a deviated trachea. What diagnosis should the nurse
Anticipate?

.A-pleural effusion

B-tension pneumothorax

C-pneumothorax

D-cardiac tamponed

232) a nurse is caring for a patient receiving total parental


nutrition(TPN).the patient reports the sudden onset of feeling short
of breath and anxious. The nurse hears crackles in bilateral lower
lobes of the lungs and the patents O2 saturation is 90% on room air.
The nurse must IMMEDIATELY: by

A-asses the patient’s capillary blood glucose

B-attempt to suction the patient’s airway

C-notify the physician

D-stops the TPN

233) a client is receiving supplemental oxygen and the nurse is


monitoring the client oxygen saturation level using pulse oximetry
the nurse notify the physician if the client reaches

Which oxygen saturation level?

A-98%
81 ‫الصفحة‬ 0569945982 / ‫ محمد سامي جوال‬: ‫استاذ‬
B-92%

C-94%

D-96%

234)Patient Ali 60 years old in ICU diagnosed with coronary


insufficiency patient was complaining of pain The pain began 3 hours
earlier and is associated with diaphoresis and dyspnea. Examination
results are unremarkable, except for diaphoresis.A 12-lead ECG
reveals normal sinus rhythm with large R waves and Horizontal ST-
segment depression in leads V1 through V3,As a nurse what is the
cause makes ST-segment depression?

A-Ischemia

B-ventricular tachycardia

C-ventricular hyperactive

D- it’s normal reading

235)Eight weeks pregnant and multi-gravida women is in the


midwife’s clinic, requesting for an abortion. The Midwife is
discussing with her the possibilities of continuing her pregnancy,
reproductive health and the birthSpacing methods. What aspect of
care identified?

A. Ethical responsibilities

B. Patient's rights to health care

C. Secondary health care preventions

D. Legal concerns to prevents complications

236)As part of neurological assessment, which of the following is


associated with the highest score on theGlasgow Coma Scale?

A-Eye opening t pain, no verbalization

B-Confused, obey commands

82 ‫الصفحة‬ 0569945982 / ‫ محمد سامي جوال‬: ‫استاذ‬


C-Localizes pain, abnormal extension

D-Eye opening to speech, confused

237)A community health nurse visits a patient who has suffered a


stroke. The patient’s spouse explains to the nurse that the patient
chokes while eating some times. Which of the following referral
orders would The nurse anticipate needing for this patient?

A-dieticia

B-neurologist

C-physical therapist

D-speech therapist

238)When performing an abdominal assessment, the nurse should


follow which examination Sequence?

A-Inspection, auscultation, percussion, and palpation

B-Inspection, auscultation, palpation, and percussion

C-Inspection, percussion, palpation, and auscultation

D-Inspection, palpation, percussion, andauscultation

239) A nurse calls together an interdisciplinary team with members


from medicine, social service, the clergy, and nutritional services to
care for a patient with a terminal illness. Which of the following
types of Care would the team most likely be providing

A- Palliative

83 ‫الصفحة‬ 0569945982 / ‫ محمد سامي جوال‬: ‫استاذ‬


B-curative

c-respite

D-preventive

240) Cephalhematoma can cause what?

A- hyper bilirubin

241) The nurse reviews the ABG findings of a client, which reveal the
following: pH 7.20 PaCO2 35 mmHg, HCO3 22mEq/L. How does the
nurse interpret these results?

A-respiratory acidosis

B- metabolic acidosis

C-Respiratory alkalosis

D-metabolic alkalosis

84 ‫الصفحة‬ 0569945982 / ‫ محمد سامي جوال‬: ‫استاذ‬


242)Non-elastic skin. She appears pale and urea and disc r Had a
stomach illness for the past three days with Nausea and vomiting The
nurse collects and sends a bloodsample for arterial blood. Gas
analysis (see lab results). That she has Blood pressure 90/50 mmHg
Heart rate 120/min Respiratory rate26 min Temperature 39.0 ͦC
Oxygen saturation 93%

Test Result Normal value

ABG¹ HCO₃ 31 22 – 88mmol/L

ABG. PCO₂. 10.3. 4.7-6.0 kPa

PH 7.50 7.36-7.45

What is the interpretation of the ABG result?

A. Respiratory acidosis

B. Respiratory alkalosis

C. Metabolic acidosis

D. Metabolic alkalosis

85 ‫الصفحة‬ 0569945982 / ‫ محمد سامي جوال‬: ‫استاذ‬


243) This image below is

A-Papule

B-Nodule

C-Tumor

D- Wheal

244) This image below is

A-Papule

B-Nodule

C-Tumor

D-wheal

245)This image below is

A-Papule

B-Nodule

C-Tumor

D-wheal

86 ‫الصفحة‬ 0569945982 / ‫ محمد سامي جوال‬: ‫استاذ‬


246)This image below is

A-Papule

B-Nodule

C-Tumor

D-wheal

Use the picture to answer the below three questions

247) Patient with placed nephrostomy tube which of the following


sign the nurse should notify the doctor IMMEDIATELY

Low urine output

248) Priority nursing diagnosis for patient with placed nephrostomy


tube?

Risk for infection

249) which of the following statements is considered true about


nephrostomy tube :

Transient procedure

87 ‫الصفحة‬ 0569945982 / ‫ محمد سامي جوال‬: ‫استاذ‬


250) patient has depression and on antidepressants medication .while
the nurse caring him he asked ( how long the medication takes to
reach the effectiveness )The proper nurse response is?

A- 3-4 days

B- 10-4 days

C- 14-18 days

D- a month

251) What can the nurse advise the patient To prevent atelactisis?

breathing exercise

252) What is the proper terminology if the respiratory rate less than
10 R/M

A- tachypnea

B- bradypnea

253) A pregnant women in labor. What the nurse expect after the
delivery of placenta?

A-the placenta has long ublical cord attached with baby

B-the placenta has 2 arteries and one vain

254) What is secondary infertility? .

A-Men infertility

B-Both wife and husband

C-Female couldn’t get pregnant last 3 years

88 ‫الصفحة‬ 0569945982 / ‫ محمد سامي جوال‬: ‫استاذ‬


255)If there is a code red announcement in a hospital Who should
response?

A- infection control

B- the staff

C-safety

256)the nurse assigned for a Muslim patient, while caring him, the
patient was avoid touched by the nurse What should the nurse do

A- avoid touching the patient

B- try to touch him again

C- ask the patient why

257)Which of the following is one of vital signs

A- pulse

258) When the hospital calls for code blue What does that mean?

A- cardiac arrest

259)A nurse educated patient with a pacemaker and told him

A- visit doctor when batteries are dying

B- avoid carry heavy object

89 ‫الصفحة‬ 0569945982 / ‫ محمد سامي جوال‬: ‫استاذ‬


260)A patient came with a burn in his chest and faceWhat is the
priority as a nurse to do

A-give him oxygen

B- check air way

261) Patients with cardiac disease, the nurse should avoided which of
the following place when taking temperature

A-Rectal

B-Oral

C-Axillary

D-Tympanic

262)This test is primarily used for diagnosing pulmonary Embolism

A-Pulmonary angiography

263)Dashboard program in the hospitals using which type of data?

264)Where is the location of pyloric?

The pyloric sphincter is a band of smooth muscle at the junction


between the pylorus of the stomach and the duodenum of the small
intestine. It plays an important role in

90 ‫الصفحة‬ 0569945982 / ‫ محمد سامي جوال‬: ‫استاذ‬


265)If the pregnant woman in active stage, what should the nurse
avoid

A-PV

266) patient has colostomy in RT side what type of stool would the
nurse expect?

Fluid

267) Patient in surgical ward after RT hip replacement, 4 hours later


she was complaining of chest pain, dyspnea, what does the nurse
expects the patient got?

A-pulmonary Embolism

268)Patient has just diagnosed with H1N1, what is the priority action
should the nurse do?

A-isolation

269) Patient has epiglttis and the nurse should assess the responsible
nerve which is

A-hypoglossal

270)post delivery women was visiting the clinic after 3 weeks, when
the nurse assess her she did not perform palpation the uterus, what is
the best answer describe the nurse action?

A- document every thing is normal

B-inform the physician

271) a sickle cell anemic patient has sever pain and Normal vital
signs except the temperature was38, 4, what is the first intervention?

A-pain management
91 ‫الصفحة‬ 0569945982 / ‫ محمد سامي جوال‬: ‫استاذ‬
272)What is the normal range of newborn systolic blood pressure

80

273)What is the proper position should keep the mother in when


measuring the funds after delivery?

Supine position

274)A 30-year-old married female has dilatation and curettage as


therapeutic abortion. A nurse was preparing for discharge
instruction?

A-take high protein diet

B-use tampons during swimming

C-avoid sexual intercourse for two weeks

D-continue on bed rest for two weeks at home

275)Which of the following is an example of epidemic point source

A-Tuberculosis

B-Public health agency

C-Contaminated water source

276)Patient feels of muscle weakness, fatigue, can’t talks


and hypotensive. Which of the following is the diagnosis?

A-Dehydration

92 ‫الصفحة‬ 0569945982 / ‫ محمد سامي جوال‬: ‫استاذ‬


277) A 5 years old patient who underwent abdominal
surgery suffers from deficient fluid volume relatedto
nothing by mouth (NPO) status; intravenous fluid therapy is
given for dhydration. Which of thefollowing indicates that
the treatment is effective?

A-urinary out put of 15 ml/hr

B-respiration rate, 35

C-heart rate 100

D-good skin turgor

278)a patient with dementia is being treated for dehydration


the patient is confused and has been immobile for the past
month. Currently, the patient is incontinent and unable to
feed self the nursing care plan should include?

A-ambulates at least 20 steps each shift

B-coughing and deep breathing every 30 min

C-positioning and turning every 2 hrs

D-range of motion exercises to all the extremity

279)The healthcare provider is assessing the glucose level of


the a patient with a diagnosis of diabetes. Which of these is
most helpful in evaluating this patient’s long-term glucose
management

A-Fasting blood glucose level

93 ‫الصفحة‬ 0569945982 / ‫ محمد سامي جوال‬: ‫استاذ‬


B-Hemoglobin A1c

C-random blood glucose level

D-amylase and lipase

280) A grandmother 70 years old has osteoporosis fall down


the stairs and her right hip has been broken. Thewomen has
a sever pain As a nurse what type of pain do you think the
patient has?

A-Visceral pain (nociceptive)

B-Neuropathic pain

C-Deep somatic pain (nociceptive)

D-Deep bone pain

281) During the screening process for tuberculosis (TB), the


healthcare provider notes that the patient received the bacilli
Calmette-Guérin (BCG)vaccination several years ago. Which of the
following is the best method to determine this patient’s TB status?

a-sputum culture

B-chest x Ray

C-CBC

D-None of the above

282)The healthcare provider assesses the patient when the patient


changes to a sitting position , patient complains headache ,dizziness ,
blurred vision .Which of the following indication for this patient?

A-hypotension

B-Hypertension

94 ‫الصفحة‬ 0569945982 / ‫ محمد سامي جوال‬: ‫استاذ‬


C-orthostatic hypotension

D-increase intra cranial pressure

283)Which tool or scale would you use for a assess of Level of


consciousness ?

A- The Lazarus Cognitive Appraisal Scale

B- None of the above

C- Apqar score

D- Glasgow coma scale

284)a woman is evaluated for infertility, and the physician prescribes


clomiphene citrate(clomid) 50mgDaily for 5 days. The client asks the
nurse about how the medication works. Which of the following
responses by the nurse is the best?

A-clomiphene citrate(clomid) induces ovulation by changing


hormonal effects on the ovary

B-clomiphene citrate (clomid) changes the uterine lining to be more


conducive to implantation

C-clomiphene citrate (clomid) alters the vaginal PH to increase


sperm motility

D-clomiphene citrate (clomid)produces multiple pregnancy for those


who desire twins

95 ‫الصفحة‬ 0569945982 / ‫ محمد سامي جوال‬: ‫استاذ‬


285) After hemorrhoidictomy care the nurse should teach the patient
to Set in

A-sitz bath for 15-20 minutes 3tims a day

286) Why the diabetic patient rotate the insulin site when taking
subcutaneously?

287) A doctor prescribed an insulin injection for diabetic patient, as a


nurse role you have to educate her what is the angel of injection?

A-90

B-15

C-45

D-60

288)what is the primary mechanism for toddlers to learn socialize?

A-Eating

B-talking

96 ‫الصفحة‬ 0569945982 / ‫ محمد سامي جوال‬: ‫استاذ‬


C-playing

D-crying

289)Patient receiving vincristine chemotherapy medication the nurse


should know all correct except?

A-frequent check of uric acid

B-give the medication as IV

C-Bun and Creatinine rise up

D-use soft tooth brush

290)while the patient was connected to dextrose. The nurse observed


swelling on site, what do you expect as a nurse?

A-phlebitis

B-air Embolism

C-oozing

291)what is the first action of ventricular fibrillation (VF)?

A-DC shock

292)what is the proper advice for patient complaining of


constipation?

A-laxatives

293)when the nurse get a needle stick what is the first action to do?

97 ‫الصفحة‬ 0569945982 / ‫ محمد سامي جوال‬: ‫استاذ‬


A-inform the supervisor

294)A patient has undergoing for a surgery while that she was still
keeping her wedding ring as nurse what is your action?

A- call one of her relative

B-inform the security

C-put it under her pillow

D- ask her to keep it

295) Identification of community problem of mortality and morbidity


rate is?

A-assessment

B-involvement

C-evaluation

296)A patient wants to do plastic surgery and the physician told her
she should be not pregnant , what should the nurse do as priority?

A-take a consent

B-ask about last menstrual cycle

297)Care of hemorrhoidectomy at the first day post operative

98 ‫الصفحة‬ 0569945982 / ‫ محمد سامي جوال‬: ‫استاذ‬


A-give laxative and prevent constipation

B- remove dressing to monitor bleeding

298)Removal of NG tube after appendectomy?

A-listen peristalsis movement

B-if patient ask me to remove

299)After a circumcision the baby had a bleeding and infection what


should the nurse do?

A-pressure;

B-give antibiotic

300)A patient uses medication for hypertension regularly when his


BP becomes normal he stoped taking the medication, then he came to
the clinic and the BP 140/90,what should the nurse do?

301)A pregnant women has depression and the body mass index is 17
what should the nurse do?

A-reassurance

B-nutritional management

99 ‫الصفحة‬ 0569945982 / ‫ محمد سامي جوال‬: ‫استاذ‬


302) The nurse reviews the client electrolytes laboratory report and
notes That magnusim level is 2.5 mEq/L(2.5mmol/L). Which patterns
should the nurse watch for on the electrocardiogram (ECG) as a ruslt
of the laboratory value?

A-Prolonged PR interval and Wide QRS complex

B-Tall packed T Wave

C-Prominent U wave

D-wide T wave

303)The following syringe contains 2 gm of morphine sulphate, how


many grams are there in 1ml:

100 ‫الصفحة‬ 0569945982 / ‫ محمد سامي جوال‬: ‫استاذ‬


A-0.5gm

B-1gm

C-2gm

D-gm

304) The following syringe contains morphine sulphate in a


concentration of 2 mg/cc, how much total morphine this syringe

contains ?

A. 0.5 mg

B. 1mg

C. 2mg

D. 4mg

305) The following condition may developed in females due to an


increase in the production of which of the following hormones?

101 ‫الصفحة‬ 0569945982 / ‫ محمد سامي جوال‬: ‫استاذ‬


A-androgens

B-progesterone

C-estrogen

D-growth

306) Which of the following is true about gastric sleeve operation

A-Surgical procedure for decrease body weight by reduce stomach


size

307) which of the following complication for patient receiving total


Parenteral nutrition?

A-Hypoglycemia

B-Hyper glycemia

308) Patient has pyloric stenosis complain of projectile vomiting.


Which of the following signs associated with projectile vomiting?

A-Olive-shaped mass

102 ‫الصفحة‬ 0569945982 / ‫ محمد سامي جوال‬: ‫استاذ‬


309)the nurse administering a scheduled dose of pilocarpine should
place the drops in which of the following areas?

A-center of the eye ball

B-low Conjunctive sac

C-inner canthus

D-outer canthus

310) A patient who is in her first trimester of pregnancy is diagnosed


with hyperthyroidism. Which of the following post partium
complication for this patient?

A-Post partium hemorrhages

B-high body weight

C-gestational hypotension

D-all of the above

311)A 2 month patient is diagnosed with a ventricular septal deficit ,


which of the following cause of disease for this patient?

A-Mother with Folic acid deficiency during pregnancy

103 ‫الصفحة‬ 0569945982 / ‫ محمد سامي جوال‬: ‫استاذ‬


312) Which of the following religion is prohibited the blood
transfusion?

A-Christian

B-Muslims

C-Jewish

313)The nurse is caring for a client diagnosed with GERD , which of


the following most affected organs for this patient?

A- heart diseases

B-liver diseases

C-stomach and esophaguses

D-large intestine

314) Patient signed up for an umbilical hernia before the surgery the
doctor found that the patient has groin hernia too What is the doctor
action?

A- the doctor add that to consent

104 ‫الصفحة‬ 0569945982 / ‫ محمد سامي جوال‬: ‫استاذ‬


B-the doctor writing that there is another hernia could repair it in
another time

C- write another consent

315)when reviewing the health history of a patient who wants to


begin taking oral contraceptives, the nurse recalls that which
condition are contraindications to this drug therapy?

A-thrombophlebitis disorder

B-multiple sclerosis

C-hypothyroidism

316)The Appropriate Mastitis education is which from the following?

A-2-3 feeding day time and night

B-Daytime breastfeeding night pacifier

317) Patient with a chest tube, when the tube is disconnected what is
the right intervention?

A-Request for a new and replace the damaged part then reconnect
the tube

B-Return the tube back

105 ‫الصفحة‬ 0569945982 / ‫ محمد سامي جوال‬: ‫استاذ‬


C-Clamping the tube

318)Patient has dehydration , after fluid replacement how can we


know if that effectively corrected?

A-Moist mucos membrane

B-Change urin color

319)What is the Thoracentesis position?

A-Lying on unaffected side 15_30

B-Lying on affected side 15_30

C-Lying on unaffected side 30_45

D-Lying on affected side 30_45

320) What is the complications After tonsillectomy?

A-hemorrhage

B-loss hearing

321) The Weber test fork place is?

A-Temporal

B-Partial

106 ‫الصفحة‬ 0569945982 / ‫ محمد سامي جوال‬: ‫استاذ‬


C-Near ear canal

322)The DKA nursing diagnosis is?

323)A diabetic mother just delivered a normal baby with good


sucking as a nurse how can you prevent the child from hypoglycemia,
By which of the following?

A-Oral feeding

B-Good sucking

324)Where should the nurse write routine care such as oral hygiene?

A-Chick list

B-Care plan

C-Kardex

325)What is the purpose of N95 mask

A-Preventing the entrance of bacteria

B-Preventing the exist of bacteria

326)What is the proper position after hemorrhoidectomy

107 ‫الصفحة‬ 0569945982 / ‫ محمد سامي جوال‬: ‫استاذ‬


327) Immediately after a percutaneous liver biopsy, the nurse should
place the client on right side , why?

A-Prevent bleeding

328) Woman complaining from stress incontinence your advice

A-Kegel exercise

329)Patient has legs length , what can cause that?

A-Strain

B-Sprain

C-Fracture

330)The patient complain of fertility which of the following


intervention the nurse expects ordering by doctor?

A-Semen test

331) Causes of neonatal jaundice

A-increase bilirubin

332)Basal skull fracture

A- Babinski

333)Short fiber muscle

A- contracture

108 ‫الصفحة‬ 0569945982 / ‫ محمد سامي جوال‬: ‫استاذ‬


334)damage to what organ would cause central diabetes insipidus?

A-parathyroid

B-Posterior Pituitary

C-kidney

D-thyroid

335) The nurse prepares IM injection for patient with iron deficiency
anemia ,Which of the following is the best action in order to prevent
tracking of the medication?

a-Use the Z-track technique

336)A client has been diagnosed with Folic acid deficiency. The
nurse’s discharge teaching will focus on foods high in folic acid.

Which of the following foods has the highest folic acid level?

A-citrus fruits

B-Raisins

C-brewers yeast

D-Eggs

109 ‫الصفحة‬ 0569945982 / ‫ محمد سامي جوال‬: ‫استاذ‬


337)if the maintenance requirement of fluids for a child is 900ml/day

Each daily feeding account 120 ml, What is the number of feeding is
required to achieve this maintenance :

A-feedings

B-6 feedings

C-7 feedings

D-8feedings

338)a child has a third- degree burns of the hand, chest, face, which
nursing diagnosing takes priority?

A-Ineffective airway clearance

B-disturbed body image

C-impaired urinary eliminations

D-risk for infection

339)An infant how has been in foster care since birth requires a
blood transfusion. Who is authorized To give written, informed
consent for the procedure ?

A-the foster mother

B-the social worker how placed the infant In the foster home

C-the registered nurse caring for infant

110 ‫الصفحة‬ 0569945982 / ‫ محمد سامي جوال‬: ‫استاذ‬


D-the nurse-manager

340)a client who is gravida 1 Para 0 is admitted in labor, her cervix is


100%affaced, and she is dilated to 3 cm, her fetus is at +1 station.
The nurse is aware that the fetus’s head is :

A-not yet engaged

B-entering the Pelvic inlet

C-below the ischial spines

D-visible at the vaginal opening

341) The nurse observes the client csf ,Which of the following of csf
color indication for patient with bacterial meningitis?

A-Cloudy

B- Clear

C-Red

D-Brown

342) instrument used to measure intraocular pressure (glaucoma)

tonometry

345)A 40 week pregnant client is going to have their labor induced.


Which of the following medication would the nurse anticipate to give
to the client?

A-olanzapin

111 ‫الصفحة‬ 0569945982 / ‫ محمد سامي جوال‬: ‫استاذ‬


B-omeprazole

C-oxycodone

D-oxytocin

343)today a patient suffered a cardiac arrest (CPR) the manager took


Over the patient’s care, issuing orders, and expecting staff to obey
them immediately, which type of leadership did this manager exhibit:

A-Autocratic

B- Democratic

C- Bureaucratic

D- Laissez faire

344)a nurse is putting together a presentation on Meningitis.

Which of the following has not been linked to Meningitis in humans?

A-s. Pneumonia

B-H. Influenza

C-n. Meningitis

D-CL difficile

345) A 69-year-old women patient in the Medical Ward is in a semi


conscious state. Her pancreatic cancer is metastasized to her liver
and lungs and she is admitted for supportive treatment. Her
physicians discussed with the family that she will not be given the
Cardiopulmonary Resuscitation to save her life if she goes into the
cardiac arrest. Her two sons agreed but the daughter is indecisive

112 ‫الصفحة‬ 0569945982 / ‫ محمد سامي جوال‬: ‫استاذ‬


which of the following is the critical thinking behind not providing
cardiopulmonary resuscitation?
‫ االوالد موافقين‬CPR ‫االم معاها سرطان البنكرياس وفي حالة توقف قلبها ماراح يسووا لها‬
‫ افضل شي اقولها انا اخلي االم تتوفى بسالم بدون عذاب‬،‫والبنت ال‬
o A. Unilateral judgement of health professionals
o B. Refusal of patient's right to treatment
o C. Ethical dilemma and indecisiveness
o D. Mercy killing to ease suffering
346) In the Emergency Unit a nurse made an error that lead to an
admission order for the client to be on a venous thromboembolic
protocol is not processed. Two days after, a nurse notices the omitted
order for heparin 500 units subcutaneous every eight hours which of
the following statement best describes the appropriate follow-up?
o A. " I will contact the supervisor immediately about this error``
o B. " I need to contact a physician and complete a variance report``
o C. " I am too busy to complete a variance report. I'll do it next "
week”
o D. " I am so glad I didn't make that mistake, that other nurse
is going to be in trouble’’

347) A nurse manger is preparing and writing a plan for dealing


disaster(code red). Which procedure is the top priority for the nurse
manager the plan?
o A. Infection control

113 ‫الصفحة‬ 0569945982 / ‫ محمد سامي جوال‬: ‫استاذ‬


o B. Staff orientation
o C. Patient education
o D. Patient relocation
348) A nurse manager was not happy about low results of patients
survey. Which of the following is the first step for nurse manager?
o A. Schedule meeting with staff
o B. Start changes to improve
o C. Review evaluation details
o D. Report to director

349) A nurse is leading an educational session on the correct use of


oral contraceptives. One of the attendees ask the nurse what to do if
she missed taking an oral contraceptive for one day? Which of the
following should be the nurse advice to her?
o A. Continue as usual with no back up contraception
o B. Take an active pill immediately and take the next pill at
the usual time
o C. Take two pills as soon as possible and then one pill daily
at the usual time
o D. Use back up contraception such as a condom for the
next 7 days

350) A postpartum mother is to be discharged on the second day of


her forceps delivery. She had sutures on her vaginal and perineum
114 ‫الصفحة‬ 0569945982 / ‫ محمد سامي جوال‬: ‫استاذ‬
tears. She is breastfeeding her baby and eating the special food
her family. what discharge teaching needs more provided by
emphasis?
o A. Diet management and exercise plan
o B. Newborn care and vaccination records
o C. Hygiene practices and alert signs to report
o D. Family planning and child growth monitoring

351) A nurse manager assigns tasks according to clinical


competencies
Of the nurse. which of the following is the management function?
o A. Delegating
o B. Evaluating
o C. Planning
o D. Controlling

352) which type of study design provides the strongest evidence?


o A. Qualitative study
o B. Randomized control trial
o C. Systematic review of descriptive studies
o D. Systematic review of correlational studies

353) A 28-year-old women is accompanied by her mother to the


ward. She is admitted for her affective disorders with anger recent

115 ‫الصفحة‬ 0569945982 / ‫ محمد سامي جوال‬: ‫استاذ‬


events and stammering spells. She has panic followed by apathy and
listless Ness. How should the nurse manage the patient?
o A. Be supportive and use therapeutic communication
o B. Improvise sign language to control forgetfulness
o C. Maintain calm environment and avoid argument
o D. Anger management and speech therapy
354) Which of the following is most appropriate to delegate
assistance
o A. Insertion of an oral airway
o B. Changing postoperative dressing
o C. Initial interview on a newly admitted patient
o D. Repositioning a patient to side-lying position
355) A patient returns to a clinic for a follow-up visit and is
diagnosed as positive for Human Immunodeficiency Virus (HIV).
The patient expresses fear related to lack of finances, fear of social
avoidance, and hopelessness. Which of the following nursing action
should provide the most patient support?
o A. Referral to a physician infectious disease specialist
o B. Referral to a community-based HIV clinic
o C. Referral to the local public health department
o D. Recommendation to disclose the diagnosis to family

356) Which of the following represents secondary infertility?


o A. Male infertility
o B. A couple that does not conceive

116 ‫الصفحة‬ 0569945982 / ‫ محمد سامي جوال‬: ‫استاذ‬


o C. Infertility that occurs after previous pregnancy
o D. Infertility lasts for more than 3 years
357) A toddler is seen in the Emergency Room with a history of high
3 temperature for 5 days. He had watery eyes and a cough for the last
days, and then he developed a rash all over the body which started in
his face and spread down to his body. The mother did not remember
the vaccination schedule of her son. For how long the child should be
isolated?
o A. 5 days
o B. 10 days
o C. 15 days
o D. 20 days
358) The nursing director wants to evaluate the quality of nursing
care at the in-patient areas. The management team will evaluate on
quarterly basis the documentation and the relationship between the
patient's length of stay and the quality of care which of the following
is the most important data source to identify the quality of care?
o A. Patient's complaints and time taken to resolve them
o B. Patient's satisfaction level at the time of discharge
o C. Details of nursing notes for patient's progress
o D. Relation between incidents and cost of care

359) infant is born with spinal bifida

117 ‫الصفحة‬ 0569945982 / ‫ محمد سامي جوال‬: ‫استاذ‬


Which of the following complication is always found in these
Infants?
o A. Hydrocephalus
o B. Craniosynostosis
o C. Meningitis
o D. Cerebral policy
360) A nurse documented assessment on a newborn as listed (see
table)

Indicator Points
Heart rate less than 98 /min
Respiratory rate 28 /min irregular
Muscle tone Minimal flexion of the
extremities
Reflex irritability Grimace

Colour Body pink, extremities blue


What is the total Apgar Sore?
o A. 0
o B. 3
o C. 5
o D. 7

361) A 62-year-old women admitted to the emergency department for


the fourth time this year, each time the patient comes with sever

118 ‫الصفحة‬ 0569945982 / ‫ محمد سامي جوال‬: ‫استاذ‬


injuries and bruises in the body. What is your responsibility as a
nurse to prevent such incident to happen again?
o A. Reports the assault to the local police and write a report
o B. Provides information about safe shelter and support
o C. Instructs the women to move away from her home
o D. Discharge the patient to a safe shelter
362) A 5-year-old child was admitted with Nephrotic Syndrome. A
nurse noticed that the child has slight facial puffiness with mild
pitting edema on his hands and feet. there was no distended abdomen
Which type of diet the nurse should order for the child?
o A. High protein, high salt diet
o B. Low protein, low fibre diet
o C. Low protein, normal salt diet
o D. Normal protein, low salt diet
363) A nurse accidently dropped a medication ampoule, informed the
charge nurse, and completed an incident report form. The charge
nurse arranges medication replacement.
Which of the following is the immediate nursing action required?
o A. Revise protocol for medication related incidences
o B. Allocate senior nurses to medication assignments
o C. Provide missed medication dosage to patient first
o D. Incident reporting must be given the priority

119 ‫الصفحة‬ 0569945982 / ‫ محمد سامي جوال‬: ‫استاذ‬


364) A Community Nursing nurse is working with a family in their
home The parents complain that their eight year-old son is "wild"
and that he never listens to them. They become upset at his antics to
gain attention from the nurse and send him out of the room. Which
of the following responses from the nurse is the most appropriate?
o A. " That was the right thing, it teaches him who is in charge”
o B. " Don't worry, he is a boy and with time, he will grow out of
this”
o C. " Let's talk about how we can teach your child the right
behavior and your expectations of him”
o D. " You are only reinforcing his behavior when you do
that. Try putting him in time out instead”
365) While taking the history from a new patient, the nurse densified
that he had hypomanic episode which was alternating with a mirror
depressive\episode for the last two years.
what is the most likely diagnosis?
o A. Bipolar I disorder
o B. Bipolar II disorder
o C. Dysthymic disorder
o D. Cyclothymic disorder
366) The head nurse meets with staff nurses to discuss ways to
improve communication among shifts Which of the following
statement best exemplifies the final stage of conflict management?
o A. "We need to clearly define the nature of the conflict”
o B. " I will evaluate the outcomes of the strategies used monthly
o C. " Let us create a time line for the implementation of our
strategies”
o D. " I have to force you to follow the rules to resolve the issue”

120 ‫الصفحة‬ 0569945982 / ‫ محمد سامي جوال‬: ‫استاذ‬


367) A mother, who is planned for the labor induction, is started on
intravenous medication. She is in the first stage of her labor and is
having regular and increasingly stronger uterine contractions Her
cervix is1cm dilated for the past few hours; both the mother and the
baby are being monitored.
Which of the following signs should alert the midwife?
o A. Baby's head not engaged
o B. Decreasing heart rate of the baby
o C. Mother's blood pressure 110/60 mmHg
o D. Mother's perspiration and increased thirst
368) A 7-year-old child is admitted to the Emergency Department,
injury The child is oriented to the place, person and time
spontaneously obeys commands. The nurse is doing aped Coma Scale
(PGCS). Which of the following score the nurse should record?
o A. 3
o B. 8
o C. 12
o D. 15
369) A 33-year-old women has come to the outpatient clinic for
treatment of a vaginal infection. Physical assessment reveals
yellowish excessive, thin offensive and frothy discharge.
Which of the following is the most likely diagnosis?
o A. Candidiasis
o B. Trichomoniasis
o C. Bacterial vaginosis
o D. Chlamydia

121 ‫الصفحة‬ 0569945982 / ‫ محمد سامي جوال‬: ‫استاذ‬


370) A 17-years-old mother presented to the primary health after
delivery She is suffering from fatigue, anemia, fever vaginal
discharge (see lab results)

Blood pressure 81/50 mmHg


Heart rate 98 /min
Respiratory rate 26 /min
Temperature 39.6 C
Test Result. Normal Values
RBC 4.6 4.7-6.1 × 1012/L
)Male(
)Female( 4.2 - 5.4×1012/L
Hb 88 130-170 g/L (Male)
120-160 g/L (Female)
Which of the following is considered as the main maternal
postpartum hemorrhage?
o A. Death
o B. Candidacies
o C. Cervical cancer
o D. Uterine prolapse

122 ‫الصفحة‬ 0569945982 / ‫ محمد سامي جوال‬: ‫استاذ‬


371) A nurse enters the room of a patient who is on the patient
complains of not feeling well, the (see image).
What is the most likely interpretation?
o A. Sinus tachycardia
o B. Ventricular fibrillation
o C. Normal sinus rhythm
o D. Ventricular tachycardia
372) A nurse is assigned to care for a patient who Nutrition (TPN).
The patient was cyanosed. Which of the following conditions is a
TPN complications?
o A. Hyperglycemia
o B. Pneumothorax
o C. Hypervolemia
o D. Infection

123 ‫الصفحة‬ 0569945982 / ‫ محمد سامي جوال‬: ‫استاذ‬


373) A 46-year-old patient is admitted in the Female back pain which
is graded 6 on the scale of10 with the slight elevation in her blood
pressure. The eat lunch which is a low sodium diet documented that
patient is uncooperative and lunch.
What can be assessed by the nursing document patient?
o A. Subjective judgement of patient's statement
o B. Misunderstanding of patient's attitude
o C. Understatement of communication
o D. Unethical evaluation of reality

374) A nurse is caring for a 72-year-old man patient is unsteady. The


patient requests the nurse to help nurse asked the patient to wait few
minutes device to transfer him. Which of the following transfer
devices is the nurse to use?
o A. Belt
o B. Board
o C. Handle
o D. Mechanical lift
375) A 40-year-old women patient with parkinsonism Medical Ward.
The patient stated that she has the past two weeks. The nurse was
planning to Which type of diet is most suitable for parkin?
o A. Solid
o B. Liquid
o C. Semi solid
o D. Clear liquid

124 ‫الصفحة‬ 0569945982 / ‫ محمد سامي جوال‬: ‫استاذ‬


376) After teaching the deep breathing and coughing who is
undergoing a surgery, the nurse asked demonstration and then
helped him in correcting, What part of therapeutic communication is
used?
o A. Evaluation
o B. Intervention
o C. Identification
o D. Demonstration
377) A nurse is caring for patient who is being admitted tract
infection. The patient feels cold and shivering

Blood pressure 110/70 mmHg


Heart rate 110 /min
Respiratory rate 22 /min
Temperature 39.7C
Which of the following is the best nursing action?
o A. Provide a hot drink
o B. Cover the patient with light blanket
o C. Start the air conditioning system
o D. Turn off lights and close curtains
378) A nurse is assigned to care for a Muslim female emergency
cesarean section. The patient asks the to wear a head scarf during the
surgery. Which of the following is the most appropriate?
o A. Ask the patient to remove the scarf
o B. Tell the patient that it is ok to wear the scarf
o C. Ask the patient to wear a surgical cap
o D. Tell the patient that a request will be during the surgery

125 ‫الصفحة‬ 0569945982 / ‫ محمد سامي جوال‬: ‫استاذ‬


379) The nurse is assigned to care for several patients, Who should be
assessed first by the nurse?
o A. A diabetic patient who will be discharged
o B. A patient with rheumatoid arthritis who has
o C. An asthmatic patient who is due for
o D. A patient with surgical incision who required
380) A nurse working in medical unit is preparing to with droplet
precaution measures in place. The the following personal protective
equipment Eyewear, what is the correct sequence foe putting the
equipment on?
o A. Face Mask, Gown, Eyewear, and Gloves
o B. Gown, Face Mask, Eyewear, and Gloves
o C. Eyewear, Cloves, Face Mask, and Gown
o D. Gloves, Gown, Face Mask, and Eyewear
382) A post-surgery client has a normal assessment with 37.6C at
0800 hours Later in the day, client is he continues to be stable Based
on client's current temperature reading actions should be taken by a
nurse?
o A. Inform surgeon since discharge should be
o B. Record temperature twice daily, at morning
o C. Aspirin 81 mg should be administered
o D. Teach client to use a spirometer and drink recheck temperature
in two hours
383) A 33-year-old man with meningitis present Department.
Healthcare providers plan to precautions what is the best nursing
practice for this case?
o A. Use N95 mask all the time
o B. Monitor the negative air pressure
o C. Use respiratory protective equipment all the

126 ‫الصفحة‬ 0569945982 / ‫ محمد سامي جوال‬: ‫استاذ‬


o D. Keep the patients with the same diagnosis

384) A 35-year-old patient was admitted to the Emergency


Department accompanied by her include severe epigastric pain,
abdominal tender the last 24 hours. She was anorexic and had since
the last few hours. She was feeling elimination and body fluid loss.
Which of the following should the nurse do to exertion?
o A. Explain that she will feel better soon
o B. Show support and understanding
o C. Encourage her to drink fluids
o D. Help he rest undisturbed
385) A 15-year-old girl is admitted to the hospital with diarrhea. She
has been repeatedly vomiting for now weak and lethargic. She is
oriented to time to questions appropriately. The nurse prepares
temperature using an electronic thermometer. Which measurement
would be most appropriate?
o A. Oral
o B. Rectal
o C. Axillary
o D. Tympanic
386) In the hospital digital dashboard, what types of data displayed?
o A. Staffing
o B. Financial
o C. Performance
o D. Knowledge-based

127 ‫الصفحة‬ 0569945982 / ‫ محمد سامي جوال‬: ‫استاذ‬


387) A four-year-old child is seen in the Emergency Department
fracture of the left arm. The x-ray examination showed and healed
bones. What is your immediate action?
o A. Call social services to immediately arrange foster care
o B. Ask the child about the previous accidents and mana
o C. Report the child abuse to the local authorities
o D. Try to establish rapport and trust with the child’s family
388) A 9-year-old child is postoperative after tonsillectomy The nurse
should ask the parents to give the child which of food after discharge
from the hospital?
o A. Meat and rice
o B. Hot dog and potato chips
o C. Mashed potatoes and soup
o D. Cucumbers and tomato salad
389) Which of the following patients the nurse should see first?
o A. Patient complaining of muscle aches and fever
o B. Patient scheduled for electrocardiography
o C. Patient newly diagnosed with hypertension
o D. Patient with complaint of chest pain

390) When performing a postpartum assessment on a women the


presence of clots in the lochia. The nurse examines the that they are
larger than 1 cm, Which of the following nursing actions is most
appropriate?
o A. Document the findings
o B. Notify the physician

128 ‫الصفحة‬ 0569945982 / ‫ محمد سامي جوال‬: ‫استاذ‬


o C. Reassess the client in 2 hours
o D. Encourage increased intake of fluids

391) A nurse is caring for a child with traction of fractured bone


doctor has placed a reminder to maintain even and constant What
would be the most likely understanding of the nnursefo?
o A. Add or remove weights every other day
o B. Allow weights to hang free continuously
o C. Elevate head and foot of the bed alternatively when i
o D. Allow weights to hang free every 12 hours to achieve circulation
392) In determining the one minute APGAR score of a male in
assesses a heart rate of 120 beats per minute and respireper minute.
He has flaccid muscle tone with slight flex44 resistance to
straightening. He has a loud cry with stimula colour is acrocyanotic,
What is the APGAR score for the infant?
o A. 7
o B. 8
o C. 9
o D. 10
393) Which of the following is the appropriate nursing advice to
mother complaining of epigastric burning sensation and do use of
drugs?
o A. Chewing gums
o B. Increase fluids at bedtime
o C. Drink orange juice on getting up
o D. Eat small meals every 2 to 3 hours

129 ‫الصفحة‬ 0569945982 / ‫ محمد سامي جوال‬: ‫استاذ‬


394) Clomiphene citrate (Clomid) is prescribed for a 32-year-old
infertility treatment The nurse should understand that this
medication is used for following actions?
o A. Induce ovulation
o B. Decrease prolactin level
o C. Reduce endometriosis
o D. Stimulate the release of Follicle-Stimulating Hormon
395) Home Health Care Department is newly established ar a
hospital. At launch, nurses asked the charge nurse about work at this
department Which of the following is the first step of home health
care?
o A. Referral
o B. Planning
o C. Home visit
o D. Assessment
396) A 23-year-old vaginal delivery primigravida mother was
discomfort due to breast engorgement on the second post The mother
complained of pain on the breast site and the able to suck the milk.
Which of the following will relief the mother's discomfort?
o A. Breast binder
o B. Well-fitting brassiere
o C. Encourage breast feeding
o D. Lactation suppressing medication

130 ‫الصفحة‬ 0569945982 / ‫ محمد سامي جوال‬: ‫استاذ‬


397) The nurse is assessing a child who has Tetralogy of Fallot
observed that the child is having clubbing in his fingernails Which of
the flowing might be the reason for this clubbing?
o A. Prolonged tissue hypoxia
o B. Delayed physical growth
o C. Inactive bone marrow
o D. Pulmonary fibrosis
398) A 47-year-old man is diagnosed with tuberculosis (TB) and
negative pressure room Which of the following should wear a face
mask?
o A. Patient's health care providers
o B. All people who enter the patient's room
o C. Person has close contact with the patient
o D. Family members who are at risk for infection
399) a nurse is caring for a patient who had Coronary Artery bypass
Graft Surgery (CABG) four hours ago. The nurse notices that the
patient has increased confusion and is restless. The patient reports
nausea, weakness and paresthesia in the extremities (see lab results)
Test Result Normal Values

Sodium 145 134-146 mmol/L

Potassium 6.8 3.5-5.2 mmol/L


Calcium 2.50 2.15-2.62 mmol/L

Which of the following is the best medication?


o A. Naloxone (Narcan)
o B. Hydralazine (Apresoline)
o C. Potassium chloride (KCI)
o D. Sodium polystyrene sulfonate (Kayexalate)
131 ‫الصفحة‬ 0569945982 / ‫ محمد سامي جوال‬: ‫استاذ‬
400) A 13-year-old patient is admitted for diarrhea and vomiting. He
looks pale and lethargic. A nurse is preparing to give IV hypotonic
solution.

Blood pressure 110/70


Heart rate 76 /min
Respiratory rate 18 /min
Temperature 39.1 C
Which IV solution is most appropriate?
o A. 0.9% saline
o B. Lactated ringers
o C. 10% dextrose in water
o D. 0.45% sodium chloride
401) A high school girl, who has fears of being obese and abased with
being thin, visited the primary healthcare center with her mother for
counselling The mother reports that her daughter refuses to eat
during meal times with the family, and often pretends being a sleep
to skin meals. Which of the following disorders best describe girl
condition?
o A. Bulimia
o B. Obesity
o C. Substance Abuse
o D. Anorexia Nervosa

132 ‫الصفحة‬ 0569945982 / ‫ محمد سامي جوال‬: ‫استاذ‬


402) a nurse is completing the preoperative checklist for one of the
patients who ring is wearing a ring. What is the most appropriate
action?
o A. Give ring to security office
o B. Lock ring with patient's valuables
o C. Call patient's family to give them the ring
o D. Respect patient's choice and leave ring on patient's finger
403) which of the following is the most appropriate action for a head
nurse starting at a new hospital?
o A. Make immediate change at the unit
o B. Plan and coordinate new strategies
o C. Assess unit activities for at least three months
o D. Ask about the previous head nurse managerial style
404) a nurse is caring for a child with traction of fractured bone. In
the chart, a doctor has placed a reminder to maintain even and
constant traction What would be the most likely understanding of the
nurse for this order?
o A. Add or remove weights every other day
o B. Allow weights to hang free continuously
o C. Elevate head and foot of the bed alternatively when in pain
o D. Allow weights to hang free every 12 hours to achieve good
circulation

133 ‫الصفحة‬ 0569945982 / ‫ محمد سامي جوال‬: ‫استاذ‬


405) A mother brought her 6-month-old healthy infant to the well-
baby clinic Which immunization should the nurse anticipate to
administer as per World Health Organization's recommendation?
o A. Varicella (Chicken pox)
o B. Rotavirus and hepatitis
o C. Measles, Mumps, Rubella
o D. Diphtheria, Tetanus and pertussis
406) A nurse diagnosis a patient with readiness for the This diagnosis
is classified within which of the nursing diagnoses?
o A. Acute nursing diagnoses
o B. Risk nursing diagnoses
o C. Wellness nursing diagnoses
o D. Possible nursing diagnosis
407) A 59-year-old women is admitted in the Medical her arms and
legs. The muscular strength progressively decreased within one year.
She pattern and has difficulty in swallowing. Which of the following
nursing diagnosis require?
o A. Risk of choking due to disturbed swallowing
o B. Weakness and fatigue due to lower muscle
o C. Disturbed breathing due to chest muscle
o D. Disturbed activities of daily living due to

134 ‫الصفحة‬ 0569945982 / ‫ محمد سامي جوال‬: ‫استاذ‬


408) A nurse is preparing a sterile field prior the Which of the
following procedures illustrates the nurse to the principles of aseptic
technique?
o A. Touch the outer 2-inch border of the sterile gloves
o B. Place the sterile linen-wrapped package on waist level
o C. Hold the bottle lip 1-2 inches above sterile pouring the solution
o D. Open sterile item while holding outside hand then throw object
onto sterile field
409) A man is to be discharged from the General appendectomy. The
precautionary measures plans are discussed with him What is the
most important desired outcome after?
o A. Remain free of post-surgical complications
o B. Report fever, redness or drainage from
o C. Use pain management techniques apropos
o D. Resume gradual activities and avoid weight
410) A 40-year-old man. smoker, presents to the clinic On
examination, the toes are cold to the touch extremities is pale to blue.
The pedal pulse. Examination of the fingers shows small ulceration
the skin. Blood glucose testing is normal and then history of diabetes
On admission

Blood pressure 140/90 mmHg


Heart rate 86 /min
Respiratory rate 22 /min
Oxygen saturation 98% room air

Which of the following would be most effective?


o A. Antibiotic administration
o B. Reduced fat intake

135 ‫الصفحة‬ 0569945982 / ‫ محمد سامي جوال‬: ‫استاذ‬


o C. Smoking cessation
o D. Regular exercise

411) A 60-year-old man client had a permanent complains of chest


pain and dyspnea with rapid feels suffocated and appears restless

Blood pressure 100/70 mmHg


Heart rate 96 /min
Respiratory rate 32 /min
Temperature 37.2C
What is the immediate nursing intervention?
o A. Monitor and report findings of chest
o B. Chest X-ray to identify dislocation of
o C. Manage pain with medication as ordered
o D. Administer oxygen as ordered

412) A 28-year-old man and his wife involved in which his wife was
killed. The client is being Care Unit for multiple rib fractures and a
broken which room his wife is located. Which of the following is the
most appropriate?
"o A. "Your wife is not in the hospital
o B. " I'm sorry, but your wife did not survive the
o C. "I need to get your family so that you can wife”
o D. " The doctor will be talking to you about located”
413) During surgery requiring general anaesthesia and a carotid
pulse is not palpate. How many compressions per minute should be
o A. 50
o B. 60
o C. 80
o D. 100

136 ‫الصفحة‬ 0569945982 / ‫ محمد سامي جوال‬: ‫استاذ‬


414) A nurse is planning to discharge a known HIV, a the Isolation
Unit after the recovery from upper, Which of the following nursing
problem requires?
o A. Risk of infection due to altered immune
o B. Fluid volume deficit due to frequent diarrh
o C. Anxiety due to disease, fear and social
o D. Weight loss due to higher metabolism rate

415) An elderly patient who has an aortic aneurysm Intensive Care


Unit to a Medical Surgical Unit on day. While assessing the client, a
nurse notes extremities and is unable to palpate the pedal pulse
Which intervention should the nurse implement?
o A. Wrap the lower extremities with warm
o B. Use a Doppler ultrasound to reassess the
o C. Elevate the extremities above heart level
o D. Place a bed cradle over the bed to levitate
416) The head nurse of a Coronary Care Unit delegated the staff a
senior nurse in that unit What initial step must the head nurse
implement before?
o A. Check the hospital policies for delegating tasks
o B. Explain the task to the senior nurse
o C. Negotiate with the senior nurse
o D. Take the signature of the senior nurse

137 ‫الصفحة‬ 0569945982 / ‫ محمد سامي جوال‬: ‫استاذ‬


417) A 22-year-old gravida 2 para 1 with gestational age 38 week
admitted to the hospital. The chief complaint is decreased the fetal
non-stress test revealed decreased variability and fetal movement.
The next morning as part of the antenatal the nurse checks the fetal
heart rate by Doppler Sonicaid decreased the fetal heart rate to less
than 100 /min. which of the following action the nurse should do
first?
o A. Reassure the mother that the FHR is Ok
o B. Notify immediately the physician or midwife
o C. Reposition the patient to left lateral position
o D. Ask the mother about the pattern of fetal movement

418) A child was admitted to the hospital three hours ago with a
injury. The child responds appropriately, but sluggishly to drifts in
and out of sleep Which of the following best describes this patient's
level of?
o A. Lethargic
o B. Obtunded
o C. Comatose
o D. Semicomatose
419) A patient was on a course of lithium carbonate drug. During the
nurse found that he complained from nystagmus visual hallucination,
and oliguria Which of the following drug related complications best
symptoms?
o A. Overdose
o B. Mild toxicity
o C. Severe toxicity
o D. Moderate toxicity

138 ‫الصفحة‬ 0569945982 / ‫ محمد سامي جوال‬: ‫استاذ‬


420) A 65-year-old women visited the gynecological outpatient
history reveals that she had 3 pregnancies, one abortion gestational
age, had 2 normal deliveries. She smokes 20 Her complaint is that she
wets herself when she cough embarrassing for her? Which of the
following can be considered as risk factors pelvic floor muscles?
o A. Chronic coughing
o B. Diabetes mellitus
o C. Excessive spot
o D. Sedentary life style

421) After accessing patients' medical records, which behavior nurse


shows that patients confidentiality has been breached?
o A. Reviews patients medical record
o B. Read patients care plan
o C. Disclosing patients information
o D. Documents medication administered
422) A 5-month-old boy has been vomiting green colored vomit He
has intermittent abdominal pain during which he draws his chest,
turns pale and cries forcefully. On observation, the in the stool which
has a jelly-like consistency. Abdominal pal a long. tube-like mass.
There is no fever, rash nor diarrhea are hyperactive in all
quadrants?
which is the most likely form of initial treatment?
o A. Manual manipulation
o B. Surgical resection
o C. Normal saline enema

139 ‫الصفحة‬ 0569945982 / ‫ محمد سامي جوال‬: ‫استاذ‬


o D. Laparoscopy

423) A primigravida mother is assisted out of the bed a few normal


vaginal delivery. She is taken to the bathroom cleaning herself and to
pass urine. She has difficulty in commode seat and is having no urge
to urinate. Which of the following intervention is the most desired?
o A. Teach kegel's exercises
o B. Give warm sits bath first
o C. Pour warm water over vulva
o D. Identify possible perineal tears
424) a primigravid women attends at the outpatient clinic for up.
History taking revealed that she is no medical hypothyroidism. The
nurse has classified this pregnancy one. What is the most possible
complication lead to this class if?
o A. Preeclampsia
o B. Preterm labor
o C. Congenital anomalies
o D. Postpartum hemorrhage
425) Which of the following is most appropriate to delegate assistant?
o A. Insertion of an oral airway
o B. Changing postoperative dressing
o C. Initial interview on a newly admitted patient
o D. Repositioning a patient to side-lying position
426) The nurse is assessing a 2 -years-old child with Wilms surgery
Which of the following should the nurse avoid?

140 ‫الصفحة‬ 0569945982 / ‫ محمد سامي جوال‬: ‫استاذ‬


o A. Putting the child in lateral position
o B. Palpating the child's abdomen
o C. Putting the child in a private room
o D. Provide mouth hygiene 30 minutes after meal

427) The nurse manager received complaints from some nursing to


improper work distribution and discrimination How should the
nurse manager handle the situation?
o A. Investigate the complaints?
o B. Individual counselling
o C. Identify list of problems
o D. Continue to observe

428) The nurse have been teaching a new mother how to feed was
born with a cleft lip and palate before surgical repair of Which of the
following action from the mother indicate teaching has been
successful?
o A. burping the baby frequently
o B. Prevent the infant from crying
o C. Placing the baby flat during feeding
o D. Keep the infant prone following feedings
429) A women was discharged from gynecological ward after
gestational trophoblastic disease (molar pregnancy) Which of the
following is the best advice to give her?
o A. Never to fall pregnant again
o B. To request the doctor to sterilize her
o C. To consider having her uterus removed
o D. To avoid falling pregnant for at least one year

141 ‫الصفحة‬ 0569945982 / ‫ محمد سامي جوال‬: ‫استاذ‬


430) A nurse manager assigns tasks according to clinical compet
nurses which of the following is the management function?
o A. Delegating
o B. Evaluating
o C. Planning
o D. Controlling

431) An autistic child makes no eye contact, unresponsive to


continuously spins, twist and head bang Which of the following is the
priority nursing diagnosis?
o A. Risk of injury related to head banging
o B. Impaired verbal communication related to physical
o C. Personal identity disorder related to poor ego develop
o D. Social isolation related to unresponsiveness towards

432) A mother in the postnatal Ward is being provided with the her
family and she has been instructed to be on the bed members believe
that the diet with the specific home increase the milk production and
will promote her healing Which of the following intervention is the
most desired?
o A. Respect the cultural believes and encourage activities
o B. Teach and monitor for correct breast feeding practices
o C. Discuss dietary plan and monitor weight gain
o D. Explain post-partum care and follow ups
433) A nurse is performing an assessment of a women who is delivery
Which assessment finding would indicates a need to physician?
o A. Hemoglobin of 11.0 g/dl
142 ‫الصفحة‬ 0569945982 / ‫ محمد سامي جوال‬: ‫استاذ‬
o B. White blood cell count of 12,000
o C. Fetal heart rate of 180 beats per minute
o D. Maternal pulse rate of 85 beats per minute

434) A 17-years-old mother presented to the primary health after


delivery. She is suffering from fatigue, anemia, fever vaginal
discharge (see lab results)

Blood pressure 81/50 mmHg


Heart rate 98 /min
Respiratory rate 26 /min
Temperature 39.6C
Test Result Normal Values

RBC 4.6 4.7-6.1 × 1012/L


(Male)
L / 1012 × 5.4-4.2
(Female)

Hb 88 130-170 g/L (Male)

Calcium 2.50 2.15-2.62 mmol/L


g/L 160-120
(Female)

Which of the following is considered as the main maternal


postpartum hemorrhage?
o A. Death
o B. Candidacies
o C. Cervical cancer

143 ‫الصفحة‬ 0569945982 / ‫ محمد سامي جوال‬: ‫استاذ‬


o D. Uterine prolapse
435) What must be known on legal points of delegation when process
to a new nurse?
o A. Evaluation of performance of delegate by clients
o B. Actual time it takes to complete the task by delegat
o C. Institution definition of the job description of
o D. Number of times that the delegate has previously task

436) A nurse manager includes staff in decision-making process what


unit structure is used?
o A. Divisional
o B. Centralized
o C. Functional
o D. Decentralized
437) A 20-year-old primigravida, who is pregnant at 40 weeks
admitted to labor and delivery unit in active labor. Vaginal revealed
that, the fetal occiput is close to the maternal Which of the following
would the nurse expect to be increase this fetal position?
o A. Leg cramps
o B. Back discomfort
o C. Vaginal bleeding
o D. Nausea and vomiting
438) A 5-year-old child is seen in the primary care clinic with
headache, malaise for about 2 days and today he has a fluids For how
long the child should be isolated after formation of?
o A. 2 days
o B. 6 days

144 ‫الصفحة‬ 0569945982 / ‫ محمد سامي جوال‬: ‫استاذ‬


o C. 10 days
o D. 15 days

439) A 1-year-old child was seen to the Emergency Department


abdominal pain, palpable sauge-shaped mass, and Intussusception is
suspected Which of the following is the best diagnostic evaluation to?
o A. X-ray
o B. endoscopy
o C. Rectal biopsy
o D. Ultrasonography
440) A nurse in the Neonatal Intensive Care Unit is caring for
premature newborn, is diagnosed with Respiratory Distre(RDS) and
the doctor ordered administrating surfactant surfactant should be
given by which of the following routes?
o A. Intravenous
o B. Subcutaneous
o C. Intramuscular
o D. Endotracheal
441) A 69-year-old patient was discharged from instructions to have
follow-up physiotherapy sessi paralysis as a result of the brain stroke.
A few physiotherapy sessions, he showed fast recovery sadly that
despite his progress he will remain Which of the following should be
the prioritize patient?

145 ‫الصفحة‬ 0569945982 / ‫ محمد سامي جوال‬: ‫استاذ‬


o A. Support for health maintenance
o B. Increase activity and exercise
o C. Identify coping mechanism
o D. Enhance self esteem

442) A patient complains of pain when standing upright hump on the


upper back. In the past year slightly shorter. The doctor has
suggested tests to What is the best appropriate intervention?
o A. Instruct the patient in the use of prescribed magnesium
supplements
o B. Have the patient sleep propped on two pain
o C. Prepare the patient for a CT scan of both
o D. Instruct the patient in the use of vitamins

443) A 56-year-old patient was diagnosed with instructions about


administration of insulin Regular Insulin and Neutral Protamine
Hagedo subcutaneously every morning?Which of the following is
right period for(in minutes) after administration?
o A. 10-15
o B. 30-60
o C. 60-90
o D. 240-360
444) In a post-operative patient in the Surgical Unit most indicative
,of a developing complication?
o A. Increasing alertness

146 ‫الصفحة‬ 0569945982 / ‫ محمد سامي جوال‬: ‫استاذ‬


o B. Weak and rapid pulse
o C. Negative Homans' sign
o D. Minimal bowel sounds in four quadrants

445) A 40-year-old man is admitted to a Coronary pain. The ECG


has normal sinus rhythm with leads V1-V4. Blood pressure 123/69
mmHg What is the most likely nursing diagnosis?
o A. Acute chest pain
o B. Myocardial in fraction
o C. Decreased cardiac output
o D. Ineffective tissue perfusion
446) A 75 year-old bedridden patient is hosp Medical ward for the
treatment of her in Area. Her wound is infected with multiple Longer
than normal time to treat. She Crying spells and is talking about
death Which of the following nursing diagnosis must be
o A. Health and wellness
o B. Coping mechanism
o C. Self-perception
o D. Belief system
447) A 1-year-old girl admitted to pediatric medical unit significant
weight loss, diminished mid-arm circumference diarrhea, and red
hair Which of the following type of malnutrition do the nurse suggest
o A. Marasmus
o B. Spitting up

147 ‫الصفحة‬ 0569945982 / ‫ محمد سامي جوال‬: ‫استاذ‬


o C. Kwashiorkor
o D. Rickets
448) which of the following statement described the latent phase?
o A. First 3 cm of cervical dilatation
o B. From onset of labour to full dilatation
o C. When the cervix is 100% effaced
o D. Time of progress from 4 cm to 7 cm

449) Nurse manager prepares unit clinical operational plan What is


top priority in the plan?
o A. Infection control
o B. Staff orientation
o C. Quality projects
o D. Safe patient care
450) The nurse was educating a postpartum woman during discharge
about importance of breast feeding Which of the following if said by
the women, indicates the need for further education?
o A. Breast milk is nutritionally balanced
o B. Breast milk reduces the risk of infection
o C. Breast feeding promotes mother-child bonding
o D. Breast feeding prevents pregnancy
451) A multipara mother complained of small vulva with swelling
following vaginal delivery of a baby weight 3.8 Kg What is the initial
nursing action should the nurse advise the mother to perform?
o A. Apply ice pack
o B. Maintain bed rest
148 ‫الصفحة‬ 0569945982 / ‫ محمد سامي جوال‬: ‫استاذ‬
o C. Administer analgesics
o D. Encourage fluid intake
452) a new community nurse in the rural clinics has to have good
background about the health status of the community in order to
assess their needs What is the appropriate way to assess the health
status of the community?
o A. Home visit
o B. Community assembly
o C. Mass information campaign
o D. Community health assessments
453) A multiparous patient on day 1 postpartum is asking the nurse
to send her baby to the nursery so she can sleep What is the most
likely phase of psychological adaptation?
o A. Taking-in
o B. Letting-go
o C. Taking-go
o D. Letting-in

454) During Cardiopulmonary Resuscitation (CPR) for a 75-year-old


man in the Emergency Department, the doctor introduced himself as
a leader for the CPR What is the most appropriate leadership style
for this situation?
o A. Autocratic
o B. Democratic
o C. Bureaucratic
o D. Laissez-faire
455) The American Academy of pediatrics suggests that removal of
the tonsils under certain conditions Which of the following meets
these conditions?
o A. 5 times viral tonsillitis per year

149 ‫الصفحة‬ 0569945982 / ‫ محمد سامي جوال‬: ‫استاذ‬


o B. Infrequent snoring and nasal quality
o C. Three times bacterial tonsillitis per year
o D. Tonsillitis accompanied by adenoid inflammation

456) A 30-year-old women was admitted with ectopic pregnancy on


the sixth gestational week. The patient was scheduled for resection of
the involved fallopian tube with end to end anastomosis Which is the
initial nursing diagnosis for this patient?
o A. Grieving
o B. Acute pain
o C. Hyperthermia
o D. Knowledge deficit
457) The nurse was educating a postpartum woman during discharge
about importance of breast feeding Which of the following if said by
the women, indicates the need for further education?
O A. Breast milk is nutritionally Balanced
O B. Breast milk reduces the risk of infection
O C. Breast feeding promotes mother-child bonding
o D. Breast feeding prevents pregnancy
458) What is the most appropriate blood product for a patient wit
hemorrhage due to upper GI bleeding?
A. Plasma o

150 ‫الصفحة‬ 0569945982 / ‫ محمد سامي جوال‬: ‫استاذ‬


B. Whole blood
C. packed red cell
D. Serum albumin

459) When the nurse assessed the fundus of a multiparous mother


who delivered 2 hours ago, she found the following
.Level: 2 cm above the umbilicus
.Position: deviated to the right
.Consistency: Not well contracted
what is the next nursing action after massaging the fundus until it
becomes firm?
o A. Assess vital signs
o B. Increase V fluids
o C. Evacuate the bladder
o D. Ask the mother to rest
460) During a CPR of an admitted patient in cardiac arrest, a family
member tasks the unit nurse to be at the bedside and say final words
to their loved one. The nurse explains to the family member that the
scene is very disturbing and the medical team is doing its job. \the
family member still insists in witnessing the resuscitation efforts
What would be the most appropriate action by the unit nurse?
o A. Allow family member to be at bedside
o B. Wait and ask permission from team leader
151 ‫الصفحة‬ 0569945982 / ‫ محمد سامي جوال‬: ‫استاذ‬
o C. Call security to escort family member out
o D. Refuse because there is not enough space in the room
461) Five years old patient was brought to emergency room with
several bruises on his body but showed fractured right forearm. He
had no signs of pain while palpating them. He seemed scared and
didn't answer any questions asked. Why should the nurse discuss this
manager with the nurse?
A-care continuity
B-abuse role out
C-support psychological
D-management fracture
462) Mother of nine children, three of them with congenital an down
syndrome; she is a primary school graduate, with status. She is not
using any method of family planning. So Health care nurse has
referred her for counselling Which of the following must be focused
on by the community nurse to provide an effective health education?
o A. Educate regardless realistic objectives
o B. Use clear and concise language
o C. Use scientific terms during explanation
o D. Explain the negative consequences in the family
463) Mr. A attended in outpatient clinic with symptoms of
shortness of breath, diarrhea and severe respiratory distress
Which of the following is the best diagnosis of Mrs. A case?
o A. Corona virus
o B. Swine Flue
o C. Zika virus
o D. Hepatitis

152 ‫الصفحة‬ 0569945982 / ‫ محمد سامي جوال‬: ‫استاذ‬


464) There is a new community nurse assigned to work in a Makkah,
one of the most multicultural cities in Saudi Arab has to interact with
multicultural population every day What is the most important step
the nurse should do befor with the clients?
o A. Asking the client about his/her cultural background
o B. Conducting an appropriate culturological assessmen
o C. Ensuring that the client has his/her a privacy
o D. Looking at the client's file to take history

465) A 36-year-old son is the primary caregiver to his 76-year-old has


many chronic diseases and need full time assistance discussed with
the community nurse the idea of referring his of the elderly day care
centers. The nurse explains the case to such services Which of the
following elderly groups this patient belongs to this service?
o A. With busy caregivers who need an assistance
o B. Who are bored staying at home and need socializati
o C. Who have been diagnosed with Alzheimer
o D. Who want to engage in handcraft activities and art

466) A 56-year-old man with a history of COPD was rushed to the


EDepartment with chest pain, shortness of breath, fewer and a
Cough. Upon assessment , crackles can be heard over the lowobes.
The patient looks pale and lethargic (see image).

153 ‫الصفحة‬ 0569945982 / ‫ محمد سامي جوال‬: ‫استاذ‬


Blood pressure 110/70 mmHg
Heart rate 130 /min
Respiratory rate 9 /min
Temperature 38.1C
Oxygen saturation 85%

What is the most likely condition?


O A. Bronchial asthma
O B. Respiratory failure
O C. Pulmonary embolism
O D. Myocardial infraction

467) A 65-year-old man is undergoing pre-operative preparation for


ndoscope procedure in which the physician will visualize the lar and
distal part of the small bowel with a camera attached to the flexible
tube Which of the following positions is the most appropriate?
o A. Left lateral Sim's
o B. Right lateral recumbent
o C. Trendelenbreg
o D. Prone
468) A term baby boy has diagnosed with Down syndrome. Physical
examination revealed flattened nose, low set ears, upward slanting
eyes single palmer crease Which of the following is the most common
congenital anomaly associated with the this disease?
o A. Developmental dysplasia of hip (DDH)
o B. Congenital heart disease
o C. Hypospadias
o D. Pyloric stenosis

154 ‫الصفحة‬ 0569945982 / ‫ محمد سامي جوال‬: ‫استاذ‬


469)A 2-day-old newborn is admitted to the nursery. While the nurse
is Administrating oral feeding, the milk returns through the child’s
nose and Mouth and the infant become cyanotic Which of the
following condition the newborn should have
O A. Anorectal malformation
O B. Tracheoesophageal fistula
O C. Cleft lip and palate
O D. Cardiac condition

470) A 7-month-old infant seen in the Emergency Department


suffering fromepisodes of severe abdominal pain, and the infant’s
stool become like red jelly. Abdominal examination revealed palpable
sausage-shaped mass in the right upper quadrant which of the
following is the best diagnosis?
o A. Hirschsprung disease
o B. Hypertrophic pyloric stenosis
o C. Infant colic
o D. Intussuception
471) Which of the following patients the nurse should see first?
o A. Patient complaining of muscle aches and fever
o B. Patient scheduled for electrocardiography
o C. Patient newly diagnosed with hypertension

155 ‫الصفحة‬ 0569945982 / ‫ محمد سامي جوال‬: ‫استاذ‬


o D. Patient with complaint of chest pain
472) A nurse receives a telephone call from the admission office of the
hospital and is told that a patient with streptococcal meningitis will
be admitted to the Medical Unit. The nurse is planning to apply
infection control measures
for the patient Which type of isolation precaution the nurse must
observe?
o A. Droplet precautions
o B. Contact precautions
o C. Airborne precautions
o D. Standard precautions

473) A 70-year-old women with a history of hypertension companies


of lack of appetite due to bloating and constipation. The abdomen is
distended and examination shows a positive fluid wave. palpation of
the abdomen confirms guarding and tenderness over the right upper
quadrant lower eg edema 3+ and pitting bilaterally are also present.
She has kilograms since her last appointment three weeks before4.5

Blood pressure 164/92 mmHg


Heart rate 90 /min
Respiratory rate 26 /min
Temperature 37.1C
Oxygen saturation 92% on room air

What is the most likely underlying health problem?

156 ‫الصفحة‬ 0569945982 / ‫ محمد سامي جوال‬: ‫استاذ‬


o A. Hepatic congestion
o B. Pulmonary hypertension
o C. Left-sided ventricle failure
o Splenomegaly
474) A nurse is trying to secure a peripheral V access in a traumatic
patient who is deteriorating. The nurse has two failed attempts What
should the nurse do next?
o A. Activate code
o B. Use Interosseous needle
o C. Consider arterial line
o D. Call another nurse who is more experienced

475) The nurse is assessing a 65-year-old patient, who reports the


fatigue, weight loss, night sweats, and a productive cough with thrick
sputum The nurse should immediately initiate isolation precautions
for which of the following?
o A. Influenza
o B. Pertussis
o C. Bacterial pneumonia
o D. Pulmonary tuberculosis
476) A patient is being admitted to the Recovery Room following a
157 ‫الصفحة‬ 0569945982 / ‫ محمد سامي جوال‬: ‫استاذ‬
thyroidectomy. The back of the neck wound is covered with dressing
During the first 15 minutes, the patient started working and having
diarrhea. A general assessment is performed with special attention
given for the high risk for hemorrhage Where would bleeding most
likely occur?
o A. Stool
o B. Vomitus
o C. Dressing
o D. Back of neck

158 ‫الصفحة‬ 0569945982 / ‫ محمد سامي جوال‬: ‫استاذ‬


‫الصفحة ‪159‬‬ ‫استاذ ‪ :‬محمد سامي جوال ‪0569945982 /‬‬
ABG

1.Drowning Respiratory acidosis


2.Vomiting Metabolic alkalosis
3.CPR Acidosis
4.Diarrhea Metabolic acidosis
5.Ileostomy Metabolic acidosis
6.Cushing syndromes Metabolic alkalosis

160 ‫الصفحة‬ 0569945982 / ‫ محمد سامي جوال‬: ‫استاذ‬


Positioning

1. CVA (cerebral Elevate the 14. mastectomy Semi fowler position


vascular accident head with affected arm
elevated on a pillow

2. craniotomy Semi fowler 15. seizure Side lying

3. Hip surgery Keep the leg in 16. shock Trendelenburg position


abduction

4. Cast Elevate 17.Percutaneous Right side lying post


extremity to Liver biopsy procedure
prevent edema

5. Asthma Sitting position 18. Nasogastric tube High fowler position

6. Cataract Semi fowler 19.Central venous Trendelenburg position


unaffected eye catheter
7. IC Elevate the 20. vap (ventilator Head of the patient's bed
P (increase head of the bed associated raised between 30-45
intracranial pressure) pneumonia) degrees

8. Distended neck vein Elevate the 21. Endoscopy Left lateral position
head of the bed
30 degree

9. Enema Sims position 22. Dyspneic Elevate head of the bed

10. Infant sleep Supine 23. Vitrectomy Prone position


position (SIDS) position

11. Post cardiac Semi fowler 24. Epidural Supine position


catheterization puncture headache
12.Spinal pefida Prone position 25. Cleft palates Prone position
position after surgery

13. cleft lip and palate Upright 26.Post Side lying


during feeding tonsillectomy’s
position-
knee chest position
27.teratology of
fallout position

161 ‫الصفحة‬ 0569945982 / ‫ محمد سامي جوال‬: ‫استاذ‬


‫الصفحة ‪162‬‬ ‫استاذ ‪ :‬محمد سامي جوال ‪0569945982 /‬‬

You might also like